Preface

Dear readers, we have started edristi English edition as well since August, 2015. We are hopeful that it will help us to connect to the broader audience and amplify our personal bonding with each other.

While presenting Day-to-day current affairs, we are very cautious on choosing the right topics to make sure only those get the place which are useful for competitive exams perspective, not to increase unnecessary burden on the readers by putting useless materials. Secondly, we have also provided the reference links to ensure its credibility which is our foremost priority. You can always refer the links to validate its authenticity.

We will try to present the current affairs topics as quickly as possible but its authenticity is given higher priority over its turnaround time. Therefore it could happen that we publish the incident one or two days later in the website.

Our plan will be to publish our monthly PDF on very first day of every month with making appropriate modifications of day-to-day events. In general, the events happened till 30th day will be given place in the PDFs. The necessity of this is to ensure the contents factual authenticity.

Reader’s satisfaction is our utmost priority so requesting you to provide your valuable feedback to us. We will warmly welcome your appreciation/criticism given to us. It will surely show us the right direction to improve the content quality. Hopefully the current affairs PDF (from 1st

Sept. to 30th Sept.) will benefit our beloved readers.

Current affairs data will be useless if it couldn’t originate any competitive exam questions. E-

Dristi has been very successful in that direction. Almost all the questions from UPPCS and other examinations have been asked from our materials. You can verify that by matching the question papers and e-Dristi contents from yourselves.

1 http://www.edristi.in/

National State Startup Ranking 2019

Que. Which state has been named the best performer in State Startup Ranking 2019? (a) (b) Maharashtra (c) Karnataka (d) Gujarat Ans. (d) Related facts:

 On September 11, 2020, Union Minister of Commerce and Industry Piyush Goyal released the second edition of the State Startup Ranking, 2019.  The Department of Industrial Policy and Promotion (DIPP), Government of India has undertaken an exercise to assess States/UTs on key areas of interventions that are germane to a healthy Startup ecosystem.  The 2019 Ranking Framework has seven broad reform areas consisting of 30 action points ranging from institutional support, easing compliances, relaxation in public procurement norms, incubation support, seed funding support, venture funding support, and awareness and outreach.  A total of 22 states and 3 Union Territories participated in the exercise, including four states from North East India.  To establish uniformity and ensure standardization in the ranking process, States and UTs were divided into Category-X and Category-Y.  Category-X entails states and UTs with a well-established startup ecosystem.  Category-Y includes all UTs, except Delhi, and all states in North East India, except Assam.  The States and UTs were classified as: Best Performers, Top Performers, Leaders, Aspiring Leaders and Emerging Startup Ecosystems.  Gujarat was the Best performer in Category X followed by Karnata and Kerala. Uttar Pradesh and Tamil Nadu occupied the lowest positions.  Andaman and Nicobar Islands was the Best performer in Category Y. Sikkim secured the bottom place.

Reference: https://www.startupindia.gov.in/content/dam/invest- india/compendium/National_Report_09092020-Final.pdf Economic Freedom of the World Report, 2019

Question: What is the rank of India in recently launched ‘Economic Freedom of the World, 2019’ report released by Indian think tank ‘Centre for Civil Society’ (CCS) in cooperation with Canada’s Fraser Institute? (a) 79th (b) 80th (c) 93rd (d) 97th

Answer: (a) Related facts:

2 http://www.edristi.in/

 On 12th September 2019, Economic Freedom of the World, 2019 annual report was released by the Indian think tank ‘Centre for Civil Society’ (CCS) in cooperation with Canada’s Fraser Institute.  Under this report, 162 countries have been ranked for year 2017, the most recent year for which comprehensive data are available.  The index published in Economic Freedom of the World measures the degree to which the policies and institutions of countries are supportive of economic freedom.  This index is prepared on the basis of data collected in five broad areas; the size of government, legal system and property rights, sound money, and freedom to trade internationally and regulation of credit labour & business.  Top five performers are Hong Kong, Singapore, New Zealand, Switzerland and United States respectively.  The 10 lowest-rated countries are: Iraq, Republic of Congo, Egypt, Syria, Democratic Republic of Congo, Angola, Algeria, Sudan, Libya, and, lastly, Venezuela.  India is ranked 79 in this index with 6.91 points.  Among the neighboring countries of India, China is ranked 113th, Bhutan at 87th, Sri Lanka at 104th, Nepal at 110th, Bangladesh at 123rd and Pakistan at 136th.

Links: https://www.fraserinstitute.org/sites/default/files/economic-freedom-of-the-world- 2019.pdf SAMVEDNA

Question:Which initiative has been launched by National Commission for Protection of Child Rights for providing counselling and psycho-social support to children during Covid 19? (a) SAMVEDNA (b) SANHARAK (c) RAKSHAK (d) ROSHNI Answer:(a) Related facts:

 The apex child rights body NCPCR(National Commission for Protection of Child Rights) has launched a toll-free tele-counselling helpline number 1800-121-2830 for children affected during COVID-19 for providing counselling and psycho-social support.  Recognized as SAMVEDNA (Sensitizing Action on Mental Health Vulnerability through Emotional Development and Necessary Acceptance),the Toll Free Tele counseling will be in service from Monday to Saturday,10 am – 1 pm and 3 pm to 8 pm.  The tele-counselling support will cater to children all over India in various regional languages also.  Counsellors have been specially trained by the expert team of National Institute of Mental Health and Neuro-Sciences (NIMHANS) to address the needs of children in these difficult times.

Links: https://www.outlookindia.com/newsscroll/ncpcr-launches-tollfree-telecounselling- number-for-children-affected-during-covid19/1940144

3 http://www.edristi.in/

President Ram Nath Kovind give assent to Jammu and Kashmir Official Languages Bill, 2020

Question: Which has been not recognized as the official language of the Union Territory of Jammu and Kashmir? (a) Dogri (b) Urdu (c) Kashmiri (d) Sindhi Answer: (d) Related facts:

 On 27 September 2020,President Ram Nath Kovind gave assent to the Jammu and Kashmir Official Languages Bill, 2020, which was recently passed by the parliament.  The Act makes Kashmiri, Dogri, Hindi, English and Urdu as the official language of the Union Territory of Jammu and Kashmir. Earlier only Urdu and English were the official language for the Union territory.  According to the 2011 census, the present population of Jammu and Kashmir is over one crore  21 lakh. The number of people speaking Urdu in the Union Territory is very less.

Links: http://www.newsonair.nic.in/News?title=President-Ram-Nath-Kovind-give-assent-to- Jammu-and-Kashmir-Official-Languages-Bill%2C-2020&id=400866 VAIBHAV Summit

Question: Consider the following statements: (1) VAIBHAV stands for Vaishwik Bharatiya Vaigyanik Summit. (2) The VAIBHAV initiative aims to bring out the comprehensive roadmap to leverage the expertise and knowledge of global Indian researcher for solving emerging challenges. Of the above correct statement/s is/are: (a) Only 1 (b) Only 2 (c) Both 1&2 (d) None of the above Answer: (c) Related facts:

 Prime Minister Narendra Modi will virtually inaugurate the Vaishwik Bharatiya Vaigyanik (VAIBHAV) Summit scheduled to be held on 2nd of October 2020.  VAIBHAV summit is a collaborative initiative by Science and Technology and Academic Organisations of the country to enable deliberations on thought process, practices and Research and development culture with a problem solving approach for well defined objectives.  It aims to bring out the comprehensive roadmap to leverage the expertise and knowledge of global Indian researcher for solving emerging challenges.  The platform will bring the Indian Overseas and resident academicians and scientists together to evolve a structure of association.  The aim of summit is to reflect in-depth on the collaboration and cooperation instruments with academia and scientists in India.  The inauguration will be followed by month-long deliberation sessions starting from 3rd October to 25th October 2020 among researchers through webinar mode.

4 http://www.edristi.in/

Links: http://www.newsonair.com/News?title=PM-Modi-to-virtually-inaugurate-VAIBHAV- Summit-scheduled-to-be-held-on-2nd-October&id=400688

President gives assent to three farm Bills

Question: Consider the following statements: (1) The Farmers’ Produce Trade and Commerce (Promotion and Facilitation) Bill, 2020 promotes barrier-free inter-state and intra-state trade and commerce outside the physical premises of markets notified under State Agricultural Produce Marketing legislations. (2) The Essential Commodities (Amendment) Bill, 2020 removes commodities like cereals, pulses, oilseeds, edible oils, onion and potatoes from the list of essential commodities. Of the above correct statement/s is/are: (a) Only 1 (b) Only 2 (c) Both 1&2 (d) None of the above Answer: (c) Related facts:

 On 14th September 2020, three bills aimed at transformation of agriculture in the country and raising farmers’ income were introduced in Lok Sabha.The bills replace ordinances promulgated on 5th June 2020.

The Three Farm Bills:

 The Farmers’ Produce Trade and Commerce (Promotion and Facilitation) Bill, 2020  The Farmers (Empowerment and Protection) Agreement of Price Assurance and Farm Services Bill, 2020  The Essential Commodities (Amendment) Bill, 2020  Union Minister of Agriculture & Farmers’ Welfare, Rural Development & Panchayati Raj, Narendra Singh Tomar introduced The Farmers’ Produce Trade and Commerce (Promotion and Facilitation) Bill, 2020 and The Farmers (Empowerment and Protection) Agreement of Price Assurance and Farm Services Bill, 2020.  Minister of State for Consumer Affairs, Food & Public Distribution, Raosaheb Patil Danve introduced The Essential Commodities (Amendment) Bill, 2020 in the Lok Sabha.

The Farmers’ Produce Trade and Commerce (Promotion and Facilitation) Bill, 2020: Main provisions –

 The new legislation will create an ecosystem where the farmers and traders will enjoy freedom of choice of sale and purchase of agri-produce.  It will also promote barrier-free inter-state and intra-state trade and commerce outside the physical premises of markets notified under State Agricultural Produce Marketing legislations. Earlier, farmers had restrictions in selling agri-produce outside the notified APMC market yards. The farmers were also restricted to sell the produce only to registered licensees of the State Governments.

5 http://www.edristi.in/

 The farmers will not be charged any cess or levy for sale of their produce and will not have to bear transport costs.  The Bill also proposes an electronic trading in transaction platform for ensuring a seamless trade electronically.  In addition to mandis, freedom to do trading at farmgate, cold storage, warehouse, processing units etc.  Farmers will be able to engage in direct marketing thereby eliminating intermediaries resulting in full realization of price.

Doubts –

 Procurement at Minimum Support Price will stop  If farm produce is sold outside APMC mandis, these will stop functioning  What will be the future of government electronic trading portal like e-NAM

Clarification –

 Procurement at Minimum Support Price will continue, farmers can sell their produce at MSP rates, the MSP for Rabi season will be announced next week  Mandis will not stop functioning, trading will continue here as before. Under the new system, farmers will have the option to sell their produce at other places in addition to the mandis  The e-NAM trading system will also continue in the mandis  Trading in farm produce will increase on electronic platforms. It will result in greater transparency and time saving

The Farmers (Empowerment and Protection) Agreement of Price Assurance and Farm Services Bill, 2020: Main provisions –

 The new legislation will empower farmers for engaging with processors, wholesalers, aggregators, wholesalers, large retailers, exporters etc., on a level playing field. Price assurance to farmers even before sowing of crops. In case of higher market price, farmers will be entitled to this price over and above the minimum price.  It will transfer the risk of market unpredictability from the farmer to the sponsor. Due to prior price determination, farmers will be shielded from the rise and fall of market prices.  It will also enable the farmer to access modern technology, better seed and other inputs.  It will reduce cost of marketing and improve income of farmers.  Effective dispute resolution mechanism has been provided for with clear time lines for redressal.  Impetus to research and new technology in agriculture sector.

Doubts –

 Under contract farming, farmers will be under pressure and they will not be able to determine prices  How will small farmers be able to practice contract farming, sponsors will shy away from them 6 http://www.edristi.in/

 The new system will be a problem for farmers  In case of dispute, big companies will be at an advantage

Clarification –

 The farmer will have full power in the contract to fix a sale price of his choice for the produce. They will receive payment within maximum 3 days.  10000 Farmer Producer organizations are being formed throughout the country. These FPOs will bring together small farmers and work to ensure remunerative pricing for farm produce  After signing contract, farmer will not have seek out traders. The purchasing consumer will pick up the produce directly from the farm  In case of dispute, there will be no need to go to court repeatedly. There will be local dispute redressal mechanism.  The two bills were passed by by Lok Sabha on 17th September 2020 and the Rajya Sabha on 20th September 2020.

The Essential Commodities (Amendment) Bill, 2020:

 The bill seeks to remove commodities like cereals, pulses, oilseeds, edibleoils, onion and potatoes from the list of essential commodities.  This will remove fears of private investors of excessive regulatory interference in their business operations.  The freedom to produce, hold, move, distribute and supply will lead to harnessing of economies ofscale and attract private sector/foreign direct investment into agriculture sector.  The bill was passed by Lok Sabha on 15th September and Rajya Sabha on22 September, 2020.  On 27th September 2020 President Ram Nath Kovind gave assent to three farm bills passed by parliament.

Links: https://pib.gov.in/PressReleaseIframePage.aspx?PRID=1654007 PM lays foundation stone for National Highway projects in Bihar

Question: Recently where did Prime Minister Narendra Modi lay foundation stone of nine highway projects worth Rs. 14,258 crore? (a) Bihar (b) Assam (c) Arunachal Pradesh (d) West Bengal Answer (a) Related facts:

 On 21 September 2020, Prime Minister Modi laid the foundation stone of nine highway projects worth Rs. 14,258 crore in Bihar.  These highway projects involve a road length of about 350 kilometres.  Prominent road projects are six laning of 39 km stretch of Patna Ring road, four laning of Ara-Mohania section of NH 30, Bakhtiarpur-Rajauli section of NH 31 and four laning of Narenpur-Purnea section of NH 131A.  PM Modi also inaugurated the Optical Fibre internet services where all the 45,945 villages of Bihar will be connected through Optical Fibre Internet Service. 7 http://www.edristi.in/

Links: https://pib.gov.in/PressReleseDetail.aspx?PRID=1657257 Blue Flag Beaches

Question:Which one is not included among the eight beaches of India recommended for the coveted International eco-label, the Blue flag certification? (a) Shivrajpur,Gujaratc (b) Kappad,Keralac (c) Golden beach,Odisha (d) Palolem Beach,Goa Answer: (d) Related facts:

 On 18th September 2020, Union Ministry of Environment, Forest and Climate Change (MoEFCC) announced at a virtual event that for the first time eight beaches of India are recommended for the coveted International eco-label, the Blue flag certification.  The recommendations are done by an independent National Jury composed of eminent environmentalists & scientists.  Blue Flag beaches are considered the cleanest beaches of the world. The eight beaches are Shivrajpur in Gujarat, Ghoghla in Daman&Diu, Kasarkod and Padubidri beach in Karnataka, Kappad in Kerala, Rushikonda in Andhra Pradesh, Golden beach of Odisha and Radhanagar beach in Andaman and Nicobar.

Links: https://pib.gov.in/PressReleaseIframePage.aspx?PRID=1656392#:~:text=On%20the%2 0eve%20of%20International,eco%2Dlabel%2C%20the%20Blue%20flag Ministry of Tourism : SAATHI app Question : The ministry of Tourism celebrated world tourism day on Sunday(27 Sep.2020) through a virtual event and launched the SAATHI application for the hospitality industry.What is SAATHI ? (a) an initiative of the tourism ministry with the Eastern Zonal Council (b) an initiative of the tourism ministry with the Inter-State Council (c) an initiative of the tourism ministry with the National Integration Council (d) None of the above Answer (d) Related facts

 The ministry of Tourism celebrated world tourism day on Sunday(27 Sep.2020) through a virtual event and launched the SAATHI application for the hospitality industry.  The ministry said SAATHI is an initiative of the tourism ministry with the Quality Council of India to assist the hospitality industry in continuing to operate safely and instill confidence among staff, employees and guests about the safety of the hotels and units.  Union minister of petroleum and natural gas and steel Dharmendra Pradhan was the chief guest for the event.  The app was launched by Pradhan who also launched a film ‘Pathik’, an initiative on Incredible India Tourist Facilitators Certification Programme (IITFC) and an ICPB MICE promotional film. 8 http://www.edristi.in/

 The ministry said the objective of the ICPB Mice promotional film is to communicate a positive message for welcoming events to India when competing destinations are already actively marketing their products.  Addressing the event, tourism minister Patel reiterated the vision of the prime minister asking every citizen to visit at least 15 destinations by the year 2022 to promote domestic tourism.

Quality Council of India

 With a view of propagating a culture of quality, Adil Zainulbhai, the current Chairman of QCI, has been instrumental in modifying the council and creating a young and talented workforce. Since its inception in 2015, the Project Planning & Implementation Division has focused on solving the key issues of the government with the help of a young team.  Major schemes and initiatives:  Swachh Bharat Abhiyan – QCI held surveys regarding the progress of the initiative.

 Pradhan Mantri Kaushal Vikas Yojna Link: https://economictimes.indiatimes.com/industry/services/travel/ministry-of-tourism- launches-saathi-app-for-hospitality-industry-on-world-tourism- day/articleshow/78350548.cms Destination North East-2020

Question : With reference to ‘Destination North East-2020’ which of the following statements is/are correct ?

1. Union Home Minister Amit Shah is the Inaugurator of the event 2. Theme for Destination North East 2020 is “The Emerging Delightful Destinations” Select the correct answer from the code given below Codes : (a)1 only (b)2 only (c)both 1 and 2 (d)Neither 1 nor 2 Answer (c) Related facts

 The Union Minister for Home Affairs Shri Amit Shah will inaugurate “Destination North East-2020” tomorrow through virtual medium.  He is also the chairman of the North Eastern Council.  Destination North East is a calendar event of the Ministry of Development of North Eastern Region conceptualized with the objective of taking the North East Region to other parts of the country and bringing them closer together in order to strengthen national integration.  The four-day programme will feature Audio Visual presentation of the tourist spots of the states and the region, messages from state icons and achievers, introduction to prominent local entrepreneurs and virtual exhibition of handicraft/ traditional fashion/ & local products. 9 http://www.edristi.in/

 The programme focuses mostly on Tourism and coincides with ‘World Tourism Day’ which falls on 27th September 2020.

Link: https://www.pib.gov.in/PressReleasePage.aspx?PRID=1659284 Mahalaya Question: Mahalaya festival is associated with which deity? (a) Durga (b) Shankar (c) Lakshmic (d) Ram Answer: (a) Related facts:

 On 17 September 2020, Prime Minister Narendra Modi greeted people on Mahalaya.  The day of Mahalaya marks the beginning of Devi Paksha and the end of the Pitri Paksha, according to Hindu calendar.  Pitri Paksha is a 16-day lunar period during which Hindus pay homage to their ancestors through shradh prayers and food offerings.  It is believed that on Mahalaya, Goddess Durga begins her journey to the Earth from her home in Kailash.  In Bengal, Mahalaya marks the beginning of Durga Puja festivities.  Bengali people traditionally wake up early in the morning on Mahalaya to recite hymns from the Devi Mahatmya (Chandi) scripture. Offerings to the ancestors are made in homes and at puja mandaps (temporary shrines).

Links: http://newsonair.com/News?title=PM-Modi-greets-people-on-Mahalaya&id=400065

Kosi Rail Mahasetu

Question: Consider the following statements: (1) On 18th September 2020, Prime Minister Narendra Modi dedicated the historic Kosi Rail Mahasetu to the nation. (2) The 1.9-kilometre long Kosi mega bridge will reduce the rail distance between Nirmali and Saraigarh from the present 298 kilometres to 22 kilometres. Of the above correct statement/s is/are: (a) Only 1 (b) Only 2 (c) Both 1&2 (d) None of the above Answer: (c) Related facts:

 On 18th September 2020, Prime Minister Narendra Modi dedicated the historic Kosi Rail Mahasetu (mega bridge) to the nation through video-conference.  The 1.9-kilometre long Kosi mega bridge has been completed at a total cost of 516 crore rupees.  With the construction of the bridge the rail distance between Nirmali and Saraigarh would come down from the present 298 kilometres to 22 kilometres.  This bridge is of strategic importance along the India-Nepal border.

Background:

10 http://www.edristi.in/

 In 1887, a meter gauge link was built in between Nirmali and Bhaptiahi (Saraigarh).  During the heavy flood and severe Indo Nepal earthquake in 1934, the rail link was washed away and thereafter due to meandering nature of river Kosi no attempt was made to restore this Rail link for long period.  The Kosi Mega Bridge line project was sanctioned by Government of India during 2003-04.

Other Projects Inaugurated:

 Apart from this the Prime Minister also inaugurated 12 rail projects related to passenger facilities for the benefit of the Bihar State.  These include a new railway bridge on the Kiul River, two new railway lines, 5 Electrification projects, one Electric Locomotive Shed and 3rd Line Project between Barh-Bakhtiyarpur.

Links: http://ddnews.gov.in/national/pm-modi-inaugurates-kosi-rail-mega-bridge-and-12- railway-projects-bihar

Institute of Teaching and Research in Ayurveda Bill 2020

Question: Where will the Institute of Teaching and Research in Ayurveda (ITRA) be established? (a) Jamnagar (b) Ahmedabad (c) Lucknow (d) Haridwar Answer: (a) Related facts:

 On 17 September 2020, The Institute of Teaching and Research in Ayurveda Bill 2020 has been passed by Rajya Sabha. The Bill was earlier passed in Lok Sabha on 19th March, 2020.  This paves the way to establish a state-of-the-art Ayurvedic institution called the Institute of Teaching and Research in Ayurveda (ITRA) at Jamnagar, Gujarat, and to confer the status of Institution of National Importance (INI) to it.  ITRA will be the first institution with INI status in the AYUSH Sector.  The ITRA is sought to be established by conglomerating the presently existing Ayurveda institutes at Gujarat Ayurved University campus Jamnagar.  This is a cluster of highly reputed institutions, namely, (a) Institute for Post Graduate Teaching and Research in Ayurveda, (b) Shree GulabKunverba Ayurveda Mahavidyalaya, and (c) Institute of Ayurvedic Pharmaceutical sciences, (d) MaharshiPatanjali Institute for Yoga Naturopathy Education & Research (to be made part of the Department of Swasthvritta of the proposed ITRA).

Links: http://ddnews.gov.in/national/parliament-passes-institute-teaching-and-research- ayurveda-bill-2020 : In a first, two women Navy officers to be posted on a warship

11 http://www.edristi.in/

Question : In a landmark event, two women officers have been selected to join as ‘Observers’ (Airborne Tacticians) on Indian Navy warships as part of the crew. What are the names of those women ? (a) Kumudini Tyagi & Riti Singh (b) Punita Arora & Padmavathy Bandopadhyay (c) Mitali Madhumita & Divya Ajith Kumar (d) Nivedita Choudhary & Anjana Bhaduria Answer (a) Related facts

 In a landmark event, two women officers have recently(September 2020) been selected to join as ‘Observers’ (Airborne Tacticians) on Indian Navy warships as part of the crew.  With this, Sub Lieutenant Kumudini Tyagi and Sub Lieutenant Riti Singh will be the first women officers in the Indian Navy to operate from warships.  The duo will be part of Multi-Role-Helicopters (MRH) crew. Both, Sub Lieutenant Kumudini Tyagi (hails from ) and Sub Lieutenant Riti Singh (hails from Hyderabad), are engineering graduates in computer science.  They were commissioned into the Navy in 2018.

Link: https://www.timesnownews.com/india/article/in-a-first-two-indian-navy-women-officers- set-to-join-warships-crew/655731 Global multidimensional poverty Index

QuestionConsider the following with respect to Global multidimensional poverty Index: (1) It was first developed in 2010 by Oxford Poverty and Human Development Initiative and United Nations Development Programme. (2) The dimensions of poverty range from deprivations of health facilities, education and living standards. Which is/are correct? (a) 1 only (b) 2 only (c) Both 1 and 2 (d) None of these Answer:(c)

 Context:  NITI Aayog said it is at an advanced stage for preparation of a Multidimensional Poverty Index (MPI) parameter dashboard to rank states and Union Territories, along with a State Reform Action Plan (SRAP).  Facts:  As the Nodal agency for the MPI, NITI Aayog has constituted a Multidimensional Poverty Index Coordination Committee (MPICC). The MPICC, chaired by Ms Sanyukta Samaddar, Adviser (SDG) has members from relevant Line Ministries and Departments. The committee held its first meeting on September 2.  The Global MPI is part of the government’s decision to monitor the performance of the country on 29 select global indices.  Global MPI is an international measure of multidimensional poverty covering 107 developing countries. It was first developed in 2010 by Oxford Poverty and Human

12 http://www.edristi.in/

Development Initiative and United Nations Development Programme for UNDP’’s Human Development Reports.  The Global MPI is released at the High-Level Political Forum (HLPF) on Sustainable Development of the United Nations in July, every year.  The dimensions of poverty range from deprivations of health facilities, education and living standards.  Global MPI is computed by scoring each surveyed household on 10 parameters based on -nutrition, child mortality, years of schooling, school attendance, cooking fuel, sanitation, drinking water, electricity, housing and household assets.

Link: https://indianexpress.com/article/business/niti-aayog-preparing-multidimensional- poverty-index-to-rank-states-uts- 6587120/#:~:text=NITI%20Aayog%20on%20Monday%20said,Reform%20Action%20Pl an%20(SRAP).&text=The%20dimensions%20of%20poverty%20range,facilities%2C%2 0education%20and%20living%20standards. MHA, MoD leave it to CCS to take final call on dual control over Assam Rifles Question:Which of the following is not a part of Central Armed Police Forces (CAPF): (a) Assam Rifles (b) National Security Guard (c) Sashastra Seema Bal (d) None of these Answer:(d)

 Context:  The Delhi High Court has ordered the Centre to take a call on the issue of bringing Assam Rifles out of the dual control of the Home Ministry (MHA) and the Defence Ministry (MoD).  Facts:  The Assam Rifles is one of the Central Armed Police Forces (CAPF) in India.  It was formed in 1835 as a militia called the ‘Cachar Levy’ to primarily protect British Tea estates and their settlements against tribal raids.  The force operates in the North Eastern part of India and complements the Indian Army in countering the insurgency and guarding the borders.  It is under the administrative control of the Home Ministry and operational control of the Indian Army.  Additional Fact:  There are seven CAPF in India. They are a) Assam Rifles (AR), b) Border Security Force (BSF), c) Central Industrial Security Force (CISF), d) Central Reserve Police Force (CRPF), e) Indo Tibetan Border Police (ITBP), f) National Security Guard (NSG) and g) Sashastra Seema Bal (SSB).

Link: https://www.thehindu.com/news/cities/kolkata/hc-asks-centre-to-decide-on-control-over- assam-rifles/article32538019.ece

13 http://www.edristi.in/

Rashtriya Poshan Maah

Question:With respect to Rashtriya Poshan Maah ,consider the following : (1) It was launched in 2016 by the Ministry of Women and Child Development. (2) It is being celebrated during the month of September. Which is/are correct? (a) 1 only (b) 2 only (c) Both 1 and 2 (d) None of these Answer:(b)

 Context:  The 3rd Rashtriya Poshan Maah is being celebrated during the month of September 2020.  Facts:  Rashtriya Poshan Mah: It is being celebrated under POSHAN Abhiyan (PM’s Overarching Scheme for Holistic Nourishment).  Objective: To encourage Jan Bhagidaari in order to create a Jan Andolan for addressing malnutrition amongst young children and women and to ensure health and nutrition for everyone.  Nodal Ministry: Ministry of Women and Child Development being the nodal Ministry for POSHAN Abhiyaan is celebrating it in convergence with partner Ministries and departments at National, States/UTs,Districts and grass root level.  Features: During this month activities related to nutrition awareness will be carried out by all the states/UTs up to the grass root level.  Additional Facts:  Poshan Abhiyan: It was launched in 2018 by the Ministry of Women and Child Development.  Aim: To achieve improvement in nutritional status of Children (0-6 years) and Pregnant Women and Lactating Mothers (PW&LM) in a time-bound manner and ensure attainment of malnutrition free India by 2022  Targets: To reduce stunting, under- nutrition, anaemia (among young children, women and adolescent girls) and reduce low birth weight by 2%, 2%, 3% and 2% per annum respectively.It would strive to achieve reduction in Stunting from 38.4% (NFHS-4) to 25% by 2022.

Link: https://pib.gov.in/PressReleasePage.aspx?PRID=1651827#:~:text=The%203rd%20Ras htriya%20Poshan,which%20was%20launched%20in%202018. India now got world’s 2nd largest road network

Question:Which of the following has now the second largest road network in the world being fuelled by the tremendous growth in construction of expressways and highways? (a) China (b) Brazil (c) Canada (d) India Answer:(d) Context: 14 http://www.edristi.in/

 India has now the second largest road network in the world being fuelled by the tremendous growth in construction of expressways and highways.  Facts:  Road, Transport and Highways Minister Nitin Gadkari said this while addressing the 60th annual convention of SIAM.  He informed that the current rate of road building per day has averaged to 30 Kilometre a day, with the highest being 40 Kilometre per day of highways.

Link: http://newsonair.com/Main-News- Details.aspx?id=399072#:~:text=India%20has%20now%20the%20second,SIAM%20thr ough%20video%20conference%20today. NCRB data shows On farmers and daily wagers committed suicide in 2019

Question:Consider the following statements with respect to National Crime Records Bureau (NCRB) data on accidental deaths and suicides: (1) The total number of suicides in the country decreased in 2019 from 2018. (2) Bihar and Punjab reported maximum increase in suicides in 2019 over 2018. Which is/are correct? (a) 1 only (b) 2 only (c) Both 1 and 2 (d) None of these Answer:(b) Context:

 According to the latest National Crime Records Bureau (NCRB) data on accidental deaths and suicides, 42,480 farmers and daily wagers committed suicide in 2019, an increase of about 6 % from the previous year, as the number of farmers committing suicide fell marginally while that of daily wagers increased 8 %.  Facts:  10,281 farmers committed suicide in 2019, down from 10,357 in 2018, whereas the figure for daily wagers went up to 32,559 from 30,132.  The suicide rate in the deeply stressed farming sector accounted for 7.4 % of the total suicides in the country, resulting in deaths of 5,957 farmers and 4,324 agricultural labourers.  The total number of suicides in the country increased to 139,123 in 2019 from 1,34,516 in 2018.  States and UTs which reported increase in suicides in 2019 over 2018 were Bihar (44.7%), Punjab (37.5%), Daman & Diu (31.4%), Jharkhand (25.0%), Uttarakhand (22.6%) and Andhra Pradesh (21.5%).  Among cities, the suicide rate (13.9) was higher than the national rate (10.4), with Kollam in Kerala and Asansol in West Bengal reporting the highest suicides rate of 41.2 and 37.8 respectively.  Out of the mega cities, the four metropolitan cities – Chennai (2,461), Delhi (2,423), Bengaluru (2,081) and Mumbai (1,229) have reported higher numbers of suicides. These four cities together have reported almost 36.6 % of the total suicides reported from 53 mega cities.The highest proportion of suicide victims (23.3 %) were

15 http://www.edristi.in/

educated up to matriculation while only 3.7 % were graduates and above, said the report.

Link: https://economictimes.indiatimes.com/news/politics-and-nation/ncrb-data-shows-42480- farmers-and-daily-wagers-committed-suicide-in- 2019/articleshow/77877613.cms#:~:text=8%20per%20cent.- ,According%20to%20the%20latest%20National%20Crime%20Records%20Bureau%20( NCRB)%20data,up%20to%2032%2C559%20from%2030%2C132.&text=Among%20dai ly%20wage%20earners%20who,were%20male%20and%203%2C467%20female. Index of Eight Core Industries JULY, 2020

Question : Which one of the following statement is correct regarding index of eight core industries ? (a) The Index of Industrial Production (IIP) is an index for India which details out the growth of various sectors in an economy such as mineral mining, agriculture and construction. (b) The Index of Industrial Production (IIP) is an index for India which details out the growth of various sectors in an economy such as mineral mining, electricity and manufacturing. (c) The Eight Core Industries comprise nearly 85.27% of the weight of items included in the Index of Industrial Production (IIP). (d) The current base year is 2010-2011. Answer (b) Related facts

 The Office of Economic Adviser, Department for Promotion of Industry and Internal Trade is releasing Index of Eight Core Industries for the Month of July, 2020.  The combined Index of Eight Core Industries stood at 119.9 in July, 2020, which declined by 9.6 (provisional) per cent as compared to the Index of July, 2019.  Its cumulative growth during April to July, 2020-21 was -20.5%.  The summary of the Index of Eight Core Industries is given below:  Coal- Coal production (weight: 10.33per cent) declinedby 5.7 per cent in July, 2020 over July,2019. Its cumulative index declined by 12.9per cent during April toJuly, 2020-21over corresponding period of the previous year.  Crude Oil-Crude Oil production (weight: 8.98per cent) declined by 4.9 per cent inJuly, 2020 over July,2019. Its cumulative index declined by 6.1 per cent during April toJuly, 2020-21over the corresponding period of previous year.  Natural Gas- The Natural Gas production (weight:6.88per cent) declinedby10.2 per cent in July, 2020 over July,2019. Its cumulative index declined by 14.7 per cent during April to July, 2020-21 over the corresponding period of previous year.  Refinery Products- Petroleum Refinery production (weight: 28.04per cent) declined by13.9 per cent in July, 2020 over July,2019. Its cumulative index declinedby 17.1per cent during April to July, 2020-21over the corresponding period of previous year.  Fertilizers-Fertilizers production (weight: 2.63 per cent) increased by 6.9 per cent in July,2020 overJuly,2019. Its cumulative index increasedby 3.9 per cent during April toJuly, 2020-21 over the corresponding period of previous year.

16 http://www.edristi.in/

 Steel-Steel production (weight: 17.92per cent)declinedby 16.4 per cent inJuly, 2020 over July,2019. Its cumulative index declined by 42.0per centduring April to July, 2020-21 over the corresponding period of previous year.  Cement- Cement production (weight:5.37per cent) declinedby13.5per cent inJuly, 2020overJuly,2019. Its cumulative index declinedby32.2per centduring April to July, 2020-21over the corresponding period of previous year.  Electricity- Electricity generation (weight:19.85per cent) declinedby2.3per centin July,2020over July,2019. Its cumulative indexdeclined by 12.4per cent duringApril to July, 2020-21over the corresponding period of previous year.

Link: https://pib.gov.in/PressReleasePage.aspx?PRID=1649978 41st meeting of the GST Council

Questions : Who chaired the 41st meeting of the GST Council ? (a) Nirmala Sitharaman (b) A B Pandey (c) B P Kanungo (d) M D Patra Answer (a) Related facts

 The 41st GST Council meeting took place via video conferencing and was chaired by Union FM Nirmala Sitharaman on 27th August 2020.  The shortfall for the FY 2020-21 works out to be Rs 2,35,000 crore. Out of this, Rs 97,000 crore is the shortfall due to GST implementation, whereas the rest is considered as due to COVID-19, which is an act of god.

The centre will provide a further relaxation of 0.5% in states’ borrowing limit under the FRBM Act for Rs.2.03 lakh crore. GST council GST Council is the governing body of GST having 33 members, out of which 2 members are of centre and 31 members are from 28 state and 3 Union territories with legislation. The council contains the following members (a) Union Finance Minister (as chairperson) (b) Union Minister of States in charge of revenue or finance (as member) (c) the ministers of states in charge of finance or taxation or other ministers as nominated by each states government (as member). Link: http://ddnews.gov.in/business/fm-nirmala-sitharaman-chairs-gst-council-meeting- discuss-compensation-loss-revenue-states GI Tag Etikoppaka Toys Question:GI tag Etikoppaka toys is from which of the following state? (a) Tamil Nadu (b) Kerala (c) Telangana (d) Andhra Pradesh Answer:(d) Context:

17 http://www.edristi.in/

 Union Education Minister has informed that indigenous toy-making will be introduced as a theme in Kala Utsav to encourage students to explore and develop a variety of local toys. He also called for initiating puppet show in Kala Utsav for secondary and higher secondary students.  Facts:  He cited the examples of places like Channapatna in Ramnagaram in Karnataka, Kondaplli in Krishna in Andhra Pradesh, Thanjavur in Tamil Nadu, Dhubari in Assam, and Varanasi in Uttar Pradesh which are developing as toy clusters.  The Global Toy Industry is more than 7 lakh crore rupees but India’s share is very little in this.  In the new National Education Policy, a lot of attention has been given on the impact of toys on different aspects of children’s lives. Learning while playing, learning to make toys, and visiting toy factories, have been made part of the curriculum.  Etikoppaka toys?  Etikoppaka is a small village on the banks of Varaha River in Visakhapatnam district of Andhra Pradesh. In 2017, the traditional Etikoppaka toys acquired a Geographical Indications (GI) tag.  Toys are made of soft wood and lacquer color. The way of toy making is also known as turned wood lacquer craft.

Link: http://newsonair.com/News?title=Indigenous-toy-making-to-be-introduced-as-theme-in- Kala-Utsav-encourage-students-to-explore%2C-develop-variety-of-local-toys%3A- Education- Minister&id=398605#:~:text=He%20said%2C%20indigenous%20toy%2Dmaking,secon dary%20and%20higher%20secondary%20students.

International Living Planet Report 2020

Question:Consider the following with respect to Living Planet Report 2020: (1) It is released by World Wildlife Fund. 2) One of the most threatened biodiversity globally has been freshwater biodiversity. 3) The largest wildlife population loss has been in Africa. Which is/are correct? (a) 1 and 2 (b) 2 and 3 (c) 1 and 3 (d) 1,2 and 3 Answer:(a)

 Context:  Living Planet Report 2020 has been released  Facts:  Released by: World Wildlife Fund. It is published every two years since 1998.  It is based on the Living Planet Index and ecological footprint calculations.  Key Takeaways from Living Planet Report 2020  There has been a reduction of 68% in the global wildlife population between 1970 and 2016.

18 http://www.edristi.in/

 75% of the Earth’s ice-free land surface has already been significantly altered, most of the oceans are polluted, and more than 85% of the area of wetlands has been lost during this period.  The most important direct driver of biodiversity loss has been land-use change, primarily the conversion of pristine habitats into agricultural systems.  The highest biodiversity loss due to land use change globally has been found in Europe and Central Asia at 57.9%, followed by North America at 52.5%, Latin America and Caribbean at 51.2%, Africa at 45.9% and Asia at 43%.  One of the most threatened biodiversity globally has been freshwater biodiversity.  The largest wildlife population loss has been in Latin America (94%)

Link: https://ec.europa.eu/jrc/en/science-update/wwf-living-planet-report-2020-reveals-68- drop-wildlife- populations#:~:text=WWF%20Living%20Planet%20Report%202020%20reveals%2068 %25%20drop%20in%20wildlife%20populations,- The%20numbers%20of&text=The%20World%20Wildlife%20Fund%20(WWF,years%20 (1970%2D2016). Indo-Pacific trilateral dialogue

Question:The first Indo-Pacific trilateral dialogue was held between which countries? (a) India-Australia-Japan (b) India-Japan-USA (c) India-Japan-France (d) India-Australia-France Answer:(d)

 Context:  The first Indo-Pacific trilateral dialogue was held between India, Australia and France.  Facts:  Objective: To build strong bilateral ties that the three countries share with each other and synergise their respective strengths to ensure a peaceful, secure, prosperous and rules-based Indo-Pacific Region.  Who attended it? The dialogue was co-chaired by Indian foreign secretary, secretary-general from the French ministry for Europe and foreign affairs and Australian foreign affairs secretary.  Additional Facts:  Indo-Pacific Region: It is an integrated theatre that combines the Indian Ocean and the Pacific Ocean and the land masses that surround them.Since it is primarily a maritime space, the Indo-Pacific is associated with maritime security and cooperation.

Link: https://timesofindia.indiatimes.com/india/india-france-australia-hold-first-trilateral- dialogue-with-focus-on-indo-pacific/articleshow/78023030.cms

19 http://www.edristi.in/

India Calls For Deeper Cooperation In Dealing With Terrorism At BRICS Meet

Question:Consider the following with respect to BRICS: (1) It is the acronym coined for an association of five major emerging national economies: Brazil, Russia, India, China and South Africa. (2) The 11th BRICS summit was held in 2019 in India under the theme BRICS: Economic Growth for an Innovative Future. Which is/are correct? (a) 1 only (b) 2 only (c) Both 1 and 2 (d) None of these Answer:(a) Context:

 India’s External Affairs Minister has virtually attended the foreign ministers meeting of BRICS countries.During the meet, foreign minister said that any challenges to international peace and security and violent conflict in various parts of the world should be resolved through political and diplomatic engagement.  Facts:  BRICS: It is the acronym coined for an association of five major emerging national economies: Brazil, Russia, India, China and South Africa.  Origin: The acronym “BRIC” was initially formulated in 2001 by economist Jim O’Neill, of Goldman Sach.The first BRIC Summit took place in 2009 in Russia.In 2010, South Africa was invited to join BRIC after which the group adopted the acronym BRICS.  cance: The comprises 42% of the world’s population, has 23% of the global GDP and around 17% of the world trade.  Chairmanship: The Chairmanship of the forum is rotated annually among the members, in accordance with the acronym B-R-I-C-S.  Recent Summit: The 11th BRICS summit was held in 2019 in Brasilia, Brazil under the theme BRICS: Economic Growth for an Innovative Future.

Link: https://thewire.in/external-affairs/brics-jaishankar-meeting UN Women

QuestionConsider the following statements with respect to UN Women: (1) It is the United Nations entity dedicated to gender equality and the empowerment of women and was established in 2010. (2) It is the principal global intergovernmental body exclusively dedicated to the promotion of gender equality and the empowerment of women. Which is/are correct? (a) 1 only (b) 2 only (c) Both 1 and 2 (d) None of these Answer:(a) Context:

 UN Women and the UN Development Programme(UNDP) has released a report titled “From Insights to Action: Gender Equality in the wake of COVID-19”. 20 http://www.edristi.in/

 Facts:  What the report says: The report says that the coronavirus pandemic and its fallout will likely push 47 million more women into poverty, reversing decades of progress to eradicate extreme poverty.  The projections estimated that by 2021, for every 100 men aged between 25 and 34 living in extreme poverty, there will be 118 women. The gap is further expected to widen to 121 women per 100 men by 2030.  UN Women:  It is the United Nations entity dedicated to gender equality and the empowerment of women.It was established in 2010.  Objectives:  To support inter-governmental bodies such as the Commission on the Status of Women in their formulation of policies, global standards and norms.  To hold the UN system accountable for its own commitments on gender equality including regular monitoring of system-wide progress.  Commission on the Status of Women(CSW):  It was established in 1946 as a functional commission of the United Nations Economic and Social Council(ECOSOC).It is the principal global intergovernmental body exclusively dedicated to the promotion of gender equality and the empowerment of women.  Beijing Declaration: It was a resolution adopted by the United Nations(UN) at the Fourth World Conference on Women in 1995.  It sets strategic objectives for the advancement of women and gender equality in 12 critical areas of concern like women and health, women in power and decision- making, the girl-child, women and the environment.  Convention on the Elimination of All Forms of Discrimination against Women (CEDAW):  It was adopted in 1979 by the UN General Assembly, is often described as an international bill of rights for women.It defines what constitutes discrimination against women and sets up an agenda for national action to end such discrimination.

Link: https://www.unwomen.org/en UNSC rejects Pakistan attempt to name Indians on terror list

Question:Consider the following with respect to UNSC 1267 Committee: (1) It was established as a result of resolution 1267 in 1999 and also known as the Al Qaeda and Taliban Sanctions Committee. (2) If an individual or an organisation is included in the list, it leads to countries to freeze the targeted groups assests, ban designated individuals from travelling and prevent the supply of weapons, technology and other aid. Which is/are correct? (a) 1 only (b) 2 only (c) Both 1 and 2 (d) None of these Answer:(c) Context:

 The UN Security Council blocked an attempt by Pakistan to list two Indians under a UN Security Council regime targeting international terrorism. 21 http://www.edristi.in/

 Facts:  UNSC 1267 Committee: It was established as a result of resolution 1267 (in 1999).It is also known as the Al Qaeda and Taliban Sanctions Committee.  Composition: The Committee comprises all 15 members of the UNSC which makes its decision by consensus.  Mandate: The Committee is mandated to:  oversee the implementation of the sanctions measures  designate individuals and entities who meet the listing criteria set out in the relevant resolutions among others.  Sanction measures: If an individual or an organisation is included in the list, it leads to:  countries to freeze the targeted groups or individual’s assets  ban designated individuals from travelling and  prevent the supply of weapons, technology and other aid.  Process by which people are listed under UNSC 1267:  Any member state can submit a proposal for listing an individual, group, or entity.  The 1267 Committee, which comprises all permanent and non-permanent members of the UNSC, meets as required with a notice of four working days.  Decisions on listing and de-listing are adopted by consensus.  Additional Information:  The UN Security Council  It was established by the UN Charter in 1945.It is one of the six principal organs of the United Nations.  The other 5 organs of the United Nations are—the General Assembly, the Trusteeship Council, the Economic and Social Council, the International Court of Justice and the Secretariat.  The primary responsibility of UNSC is to work to maintain international peace and security.  Members:  The Council consists of ten elected members and five permanent members who have veto power (P-5) – China, the United States, France, the United Kingdom and Russia.  The ten non-permanent members are elected on a regional basis by the General Assembly, (from five areas- five from Asia and Africa, one from Eastern Europe, two from Latin America, two from Western Europe) to serve two-year terms.  The council’s presidency is a capacity that rotates every month among its 15 members.

Link: https://www.thehindu.com/news/national/unsc-rejects-pakistan-attempt-to-list-indians- on-terror- list/article32507766.ece#:~:text=The%20UNSC%20committee%20decided%20to,Ul%2 DAhrar%20(JuA)%20to Togo is first African country to end sleeping sickness

Question:Consider the following with reference to Sleeping Sickness or Human African trypanosomiasis: (1) It is a water-borne parasitic disease caused by infection with protozoan parasites 22 http://www.edristi.in/ belonging to the genus Trypanosoma. (2) WHO has declared that Congo has become the first country in Africa to eliminate sleeping sickness. Which is/are correct? (a) 1 only (b) 2 only (c) Both 1 and 2 (d) None of these Answer:(d) Context:

 The World Health Organization(WHO) has declared that Togo has become the first country in Africa to eliminate human African Trypanosomiasis or sleeping sickness.  Facts:  Sleeping Sickness or Human African trypanosomiasis: It is a vector-borne parasitic disease.  Caused by: It is caused by infection with protozoan parasites belonging to the genus Trypanosoma.  Transmission: They are transmitted to humans by tsetse fly (Glossina genus) bites which have acquired their infection from human beings or from animals harbouring human pathogenic parasites.  Endemic: The disease is currently found in 36 sub-Saharan African countries.According to WHO, If untreated, the disease can become fatal.  Togo:  It is a country in West Africa bordered by Ghana to the west, Benin to the east and Burkina Faso to the north.The country extends south to the Gulf of Guinea where its capital Lomé is located.

Link: https://www.who.int/news-room/detail/27-08-2020-togo-is-first-african-country-to-end- sleeping-sickness-as-a-public-health-problem#:~:text=Mahoudeau%2FMSF- ,Togo%20is%20first%20African%20country%20to%20end,as%20a%20public%20healt h%20problem&text=Togo%20has%20received%20validation%20from,Africa%20to%20r each%20this%20milestone. Japan’s PM resigns

Question:Consider the following with respect to Ulcerative colitis: (1) It is an inflammatory bowel disease (IBD) that causes long-lasting inflammation and ulcers (sores) in a person’s digestive tract. (2) Symptoms may include Diarrhea, often with blood or pus, Abdominal pain and cramping, Rectal pain, Rectal bleeding etc. Which is/are correct? (a) 1 only (b) 2 only (c) Both 1 and 2c (d) None of these Answer:(c) Context:

 Japanese PM, 65-year-old Shinzo Abe resigned a year before he would have completed his term (September 2021) in office after the ulcerative colitis, an inflammatory bowel disease, that he has been battling for many years had worsened recently.

23 http://www.edristi.in/

 Facts:  Ulcerative colitis is an inflammatory bowel disease (IBD) that causes long-lasting inflammation and ulcers (sores) in a person’s digestive tract. Ulcerative colitis affects the innermost lining of large intestine (colon) and rectum.  The colon’s lining develops small ulcers which produce pus and mucus, leading to abdominal pain, discomfort and urge to frequently empty the colon.  Symptoms usually develop over time, rather than suddenly. Signs and symptoms may include: Diarrhea, often with blood or pus, Abdominal pain and cramping, Rectal pain, Rectal bleeding — passing small amount of blood with stool and Urgency to defecate.  Ulcerative colitis can be debilitating and can sometimes lead to life-threatening complications.  While it has no known cure, treatment can greatly reduce signs and symptoms of the disease and even bring about long-term remission.

Link: https://www.timesnownews.com/health/article/faqs-on-what-is-ulcerative-colitis/644406

Economics RBI draft on Rupee interest rate derivatives Question : The Reserve Bank on Tuesday proposed allowing foreign portfolio investors (FPIs) to undertake exchange-traded rupee interest rate derivatives transactions subject to an overall ceiling of — (a) Rs 5,000 crore. (b) Rs 7,000 crore. (c) Rs 8,000 crore. (d) Rs 9,000 crore. Answer (a) Related facts

 The Reserve Bank on Tuesday(15 September 2020) proposed allowing foreign portfolio investors (FPIs) to undertake exchange-traded rupee interest rate derivatives transactions subject to an overall ceiling of Rs. 5,000 crore.  Interest Rate Derivatives (IRD) are contracts whose value is derived from one or more interest rates, prices of interest-rate instruments, or interest rate indices.  The proposed Rupee Interest Rate Derivatives (Reserve Bank) Directions, 2020 are aimed at encouraging higher non-resident participation, enhance the role of domestic market makers in the offshore market, improve transparency, and achieve better regulatory oversight, according to the RBI.  FPIs may transact in permitted exchange-traded IRDs subject to the conditions that, at any point in time “the net long position of FPIs, collectively, and across all exchanges, in exchange-traded IRDs shall not exceed Rs 5,000 crore”, the draft regulations suggested.  Also, the net short position of an FPI on exchange-traded IRDs should not exceed its long position in government securities and other rupee debt securities.  The draft further said for the purpose of offering Rupee IRD contracts to a user, the market-maker (entities which provide bid and offer prices to users in order to provide liquidity to the market) should classify the user either as a retail user or as a non- retail user.

24 http://www.edristi.in/

 Non-retail users, as per the draft, are entities regulated by RBI, SEBI, IRDAI or PFRDA; resident companies with a minimum net worth of Rs 500 crore; and non- residents, other than individuals.  “Any user who is not eligible to be classified as a non-retail user shall be classified as a retail user,” it said.

Link: https://www.financialexpress.com/industry/banking-finance/rbi-issues-draft-rupee- interest-rate-derivatives/2084058/ MMTC-PAMP launches buy-back, exchange offers for gold

Question : Bullion refiner MMTC-PAMP has recently launched a buy-back and exchange offer for gold in the national capital in a move to help the pandemic- affected people offset their economic hardships.Which type of gold bars include in this offer ? (a) a gold bar with 9999, 999 and 995 purity, (b) a gold bar with 9999, 899 and 895 purity, (c) a gold bar with 8999, 999 and 995 purity, (d) a gold bar with 9999, 699 and 695 purity, Answer (a) Related facts

 Bullion refiner MMTC-PAMP has recently(September 2020) launched a buy-back and exchange offer for gold in the national capital in a move to help the pandemic- affected people offset their economic hardships.  For a nominal transaction fee, sellers can receive the maximum value of gold as a direct bank transfer or in the form of a gold bar with 9999, 999 and 995 purity.  Starting with its Lajpat Nagar centre in Delhi, this facility will be rolled out soon across the country.

Background

 “These are indeed challenging times worldwide, and the significant uptick in jewellers and consumers selling gold assets to offset hardships is expected to continue for some time,” MMTC-PAMP Chief Executive Officer Managing Director Vikas Singh said in a statement.  MMTC-PAMP said its on-site X-ray fluorescence technology (XRF) machines and can process weights from 10-grams to 2-kg per visit.  The procedure will take place within 60 minutes and is fully transparent as sellers can view the entire valuation process in real-time via CCTV monitors. The gold jewellery is authenticated and weighed, and upon agreement, melted and cast to form a bar, it added.

India’s demand for gold India’s demand for gold reached 690.4 tonnes in 2019. India’s gems and jewellery export stood at US$ 29.01 billion in FY20P. During the same period, India exported US$ 18.66 billion worth of cut and polished diamonds, thereby contributing 64 per cent of the total gems and jewellery export. 25 http://www.edristi.in/

Link: https://economictimes.indiatimes.com/markets/commodities/news/mmtc-pamp- launches-buy-back-exchange-offers-for-gold/articleshow/78333892.cms EASE Banking Reforms Index

Question:Parameters to rank Public Sector Banks by EASE Banking Reforms Index includes which of the following: (1) Responsible banking (2) Customer Responsiveness (3) Credit off take (4) Deepening Financial Inclusion and Digitization Which is/are correct? (a) 1 and 3 (b) 3 and 4 (c) 1,2 and 4 (d) 1,2,3 and 4 Answer:(d)

 Context:  Union Finance Minister has felicitated the best performing banks on EASE Banking Reforms Index.  Facts:  EASE Banking Reforms Index: EASE stands for ‘Enhanced Access and Service Excellence’.It was launched in 2018 with the aim to institutionalize clean and smart banking.  EASE 2.0 further progresses the reforms built in EASE 1.0 by making the reforms journey irreversible, strengthening processes and systems and driving outcomes.  Prepared by: The index is prepared by the Indian Banking Association (IBA) and is commissioned by the Finance Ministry.  Parameters: It ranks Public Sector Banks across six themes namely a) Responsible banking b) Customer Responsiveness c) Credit off take d) PSBs Udyamitra for MSMEs e) Deepening Financial Inclusion and Digitization and f) Ensuring Outcomes- Governance/HR.  Additional Fact:  Doorstep Banking Services by Public Sector Banks(PSBs): It is a part of EASE (enhanced access and service excellence) reforms that aims to provide convenience of banking services to the customers at their doorstep.

Link: https://pib.gov.in/PressReleaseIframePage.aspx?PRID=1652723#:~:text=Bank%20of% 20Baroda%2C%20State%20Bank,the%20EASE%202.0%20Index%20Results. ADB & India Sign $500 Million Loan for Delhi- RRTS Corridor

Question:The Government of India signed a $500 million loan with whom, to build a modern, high-speed 82-kilometer Delhi-Meerut Regional Rapid Transit System corridor? (a) World Bank (b) IMF (c) Asian Infrastructure Investment Bank (d) Asian Development Bank Answer:(d)

 Context:

26 http://www.edristi.in/

 The Asian Development Bank (ADB) and the Government of India signed a $500 million loan, the first tranche of a total $1 billion facility, to build a modern, high- speed 82-kilometer Delhi-Meerut Regional Rapid Transit System (RRTS) corridor.  Facts:  It will improve regional connectivity and mobility in India’s national capital region (NCR).  The first tranche loan will support construction of the first of three priority rail corridors planned under the NCR Regional Plan 2021 to connect Delhi to other cities in adjoining states.  ADB is committed to achieving a prosperous Asia and the Pacific, while sustaining its efforts to eradicate extreme poverty. Established in 1966, it is owned by 68 members—49 from the region.

Link: https://pib.gov.in/PressReleaseIframePage.aspx?PRID=1652290 National Sample Survey

Question:National Sample Survey (July 2017-June 2018) has released a report titled “household social consumption related to health”.Consider the following with respect to it: (1) About 97% of children across the country received at least one vaccination mostly BCG and/or the first dose of OPV at birth. (2) The vast majority of these vaccinations were carried out in private health facilities. Which is/are correct? (a) 1 only (b) 2 only (c) Both 1 and 2 (d) None of these Answer:(a)

 Context:  National Sample Survey (July 2017-June 2018) has released a report titled “household social consumption related to health”.  Facts:  Key Highlights of the report:  Full Immunisation: Across the country, only 59.2% of children under five years are fully immunised.  Full immunisation means that a child receives a cocktail of eight vaccine doses in the first year of life: the BCG vaccine against tuberculosis; the measles vaccine; the oral polio vaccine (OPV) , DPT/pentavalent vaccine to protect a child from diphtheria among others.  About 97% of children across the country received at least one vaccination mostly BCG and/or the first dose of OPV at birth.However, only 67% of children are protected against measles.Only 58% got their polio booster dose while 54% got their DPT booster dose.  State wise: Manipur (75%), Andhra Pradesh (73.6%) and Mizoram (73.4%) recorded the highest rates of full immunisation.  On the other hand, Nagaland has only 12% children who received all vaccinations followed by Puducherry(34%) and Tripura(39.6%).

27 http://www.edristi.in/

 Government Hospitals: The vast majority of these vaccinations 95% in rural India and 86% in cities were carried out in government health facilities and primary health centres.

Link: https://economictimes.indiatimes.com/topic/NSO-survey RBI sets sectoral norms for resolution of COVID-19 related stressed assets

QuestionThe Reserve Bank of India specified five financial ratios and sector- specific thresholds for resolution of COVID-19-related stressed assets in 26 sectors, Which of the following belongs to those financial matrics: (1) total outstanding liabilities/ adjusted tangible net worth. (2) current ratio. (3) average debt service coverage ratio. Which is/are correct? (a) 1 and 2 (b) 2 and 3 (c) 1 and 3 (d) 1,2 and 3 Answer:(d)

 Context:  The Reserve Bank of India (RBI) specified five financial ratios and sector-specific thresholds for resolution of COVID-19-related stressed assets in 26 sectors, including auto components, aviation and tourism.  Facts:  The circular issued by the Reserve Bank for resolution of the stressed assets is based on the recommendations of the K.V. Kamath Committee, which submitted its report on 4th of September.  The 26 sectors specified by the RBI include automobiles, power, tourism, cement, chemicals, gems and jewellery, logistics, mining, manufacturing, real estate, and shipping among others.  Five financial metrics need to be taken into account while deciding on a recast plan:  total outstanding liabilities/ adjusted tangible net worth,  total debt,  current ratio,  debt service coverage ratio, and  average debt service coverage ratio.  For each of these parameters, RBI has prescribed either a floor or a ceiling.  The committee sets 180 days to implement the plan and makes an inter creditor agreement (ICA) mandatory. The tenure of a loan may be extended by a maximum of two years, with or without a moratorium, the panel has said.  The resolution process shall be treated as invoked once lenders representing 75% by value and 60% by number agree to invoke the same.

Link: https://www.thehindu.com/business/Economy/rbi-sets-sectoral-norms-for-resolution-of- covid-19-related-stressed-assets/article32566248.ece

28 http://www.edristi.in/

Business Reform Action Plan 2019

Question:Consider the following with respect to Business Reform Action Plan 2019: (1) The ranking of States based on the implementation of Business Reform Action Plan started in the year 2014. (2) The Department for Promotion of Industry and Internal Trade (DPIIT) conducts the exercise for all States and Union Territories under the BRAP. Which is/are correct? (a) 1 only (b) 2 only (c) Both 1 and 2 (d) None of these Answer:(b)

 Context:  Union Minister of Finance and Corporate Affairs has announced the 4th edition of Business Reform Action Plan (BRAP) ranking of states,2019.  Facts:  Business Reform Action Plan(BRAP): The ranking of States based on the implementation of Business Reform Action Plan started in the year 2015.Till date, State Rankings have been released for the years 2015, 2016 and 2017-18.  Who Conducts the exercise? The Department for Promotion of Industry and Internal Trade(DPIIT) conducts the exercise for all States and Union Territories under the BRAP.  Objective: To attract investments and increase Ease of Doing Business in each State by introducing an element of healthy competition through a system of ranking states based on performance in the implementation of BRAP.  Parameters: It includes 180 reform points covering 12 business regulatory areas such as Access to Information, Single Window System, Labour, Environment among others.

Link: https://pib.gov.in/PressReleasePage.aspx?PRID=1651598 Crisil : Projections of Indian gdp growth rate Question : Crisil is an international rating agency.Recently(September 2020) it projects _ gdp contraction in 2020-21. (a) nine percent (b) ten percent (c) eight percent (d) seven percent Answer (a) Related facts

 The Indian economy will contract by 9 per cent in 2020-21 as the coronavirus infections are yet to peak and the government is not providing adequate direct fiscal support, ratings agency Crisil said on Thursday(10 September 2020).

29 http://www.edristi.in/

 In May, Crisil had estimated the economy to contract by 5 per cent. The latest projection comes days after official data for the June quarter showed a contraction of 23.9 per cent in the economy.

Crisil said the 9 per cent contraction will be the highest since the 1950s. The government had announced a Rs 20 lakh crore relief package but the actual new spending was less than 2 per cent of GDP. “With the pandemic”s peak not yet in sight and the government not providing adequate direct fiscal support, the downside risks to our earlier forecast have materialised,” the agency said. “A stretched fiscal position has constrained the government from spending more to support the economy. Till date, the policy push to growth remains muted, except in pockets. Our May forecast had assumed additional direct fiscal support of 1 per cent of GDP, which has not come through,” the agency said. It added that if the pandemic were to peak out in September-October, GDP growth could move into mild positive territory towards the end of this fiscal. India has overtaken Brazil as the second worst hit country with over 42 lakh coronavirus cases. Even as many analysts have been pitching for a sharp rebound or a v-shaped recovery, Crisil said the pandemic will leave a “permanent scar”. “We expect a permanent loss of 13 per cent of real GDP over the medium term,” it said, pegging its value in nominal terms at Rs 30 lakh crore and added that this is much higher than a 3 per cent permanent hit to GDP in Asia-Pacific economies estimated by its parent S&P. Link: https://www.outlookindia.com/newsscroll/crisil-projects-9-pc-gdp-contraction-in- fy21says-govt-not-providing-adequate-direct-fiscal-support/1933142 RBI revises priority sector lending guidelines

Question:Consider the following with respect to Priority Sector Lending guidelines: (1) Priority sector loans refer to loans that banks need to mandatorily lend to economically weaker sections of the society. (2) For improvement of health infrastructure, credit limit for health infrastructure (including those under ‘Ayushman Bharat’) has been trippled. Which is/are correct? (a) 1 only (b) 2 only (c) Both 1 and 2 (d) None of these Answer:(a)

 Context:

30 http://www.edristi.in/

 Reserve Bank of India has revised Priority Sector Lending guidelines to address regional disparities in the flow of priority sector credit.  Facts:  In the revised guidelines, higher weightage have been assigned to incremental priority sector credit in identified districts where priority sector credit flow is comparatively low.  Bank finance to start-ups up to 50 crore rupees, loans to farmers for installation of solar power plants for solarisation of grid connected agriculture pumps and loans for setting up Compressed Bio Gas (CBG) plants have been included as fresh categories eligible for finance under priority sector.  The targets prescribed for “small and marginal farmers” and “weaker sections” are being increased in a phased manner.  Loan limits for renewable energy have been increased under the revised guidelines.  For improvement of health infrastructure, credit limit for health infrastructure (including those under ‘Ayushman Bharat’) has been doubled.  Additional Fact:  Priority Sector lending(PSL): It means those sectors which the Government and RBI consider as important for the development of the basic needs of the country and are to be given priority over other sectors.The banks are mandated to encourage the growth of such sectors with adequate and timely credit.  Under this, Commercial banks including foreign banks are required to mandatorily earmark 40% of the adjusted net bank credit for priority sector lending.  Regional rural banks and small finance banks will have to allocate 75% of adjusted net bank credit to PSL.

Link: https://economictimes.indiatimes.com/markets/stocks/news/rbi-revises-priority-sector- lending- guidelines/articleshow/77927451.cms#:~:text=NEW%20DELHI%3A%20The%20Reserv e%20Bank,line%20with%20emerging%20national%20priorities.&text=in%20May%2020 18.- ,RBIs%20said%20the%20new%20guidelines%20will%20address%20regional%20dispa rities,flow%20of%20priority%20sector%20credit. India attends BIMSTEC meet to discuss deepening cooperation among members

Question:Consider the following with respect to Bay of Bengal Initiative for Multi- Sectoral Technical and Economic Cooperation (BIMSTEC): (1) It is a regional organization founded in 1997 through the Tashkent Declaration. (2) It comprises seven Member States: five deriving from South Asia, including Bangladesh, Bhutan, India, Nepal, Sri Lanka, and two from Southeast Asia, including Myanmar and Thailand. Which is/are correct? (a) 1 only (b) 2 only (c) Both 1 and 2 (d) None of these Answer:(b) 31 http://www.edristi.in/

Context:

 India has attended a Senior Officials Meeting(SOM) of BIMSTEC under the chairmanship of Sri Lanka to discuss ways to deepen cooperation among the member nations and holding of the next summit of the grouping.  Facts:  Bay of Bengal Initiative for Multi-Sectoral Technical and Economic Cooperation (BIMSTEC):  It is a regional organization founded in 1997 through the Bangkok Declaration.  Aim: To create an enabling environment for economic development; accelerate social progress and promote collaboration on matters of common interest in the region.  Members: It comprises seven Member States: five deriving from South Asia, including Bangladesh, Bhutan, India, Nepal, Sri Lanka, and two from Southeast Asia, including Myanmar and Thailand.  Headquarters: Dhaka, Bangladesh.  First Summit: The First Summit Meeting of the Heads of the BIMSTEC Countries was held in Bangkok, Thailand in 2004.  The Fourth Summit was held in Kathmandu,Nepal in 2018.

Link: https://www.business-standard.com/article/economy-policy/india-attends-bimstc-meet- to-discuss-deepening-cooperation-among-members-120090201799_1.html

Scientific First time a Planet found in a Galaxy other than Milky-Way

Question: Astronomers found a strong evidence of a planet by observing the X- ray data obtained from a source in the Whirlpool galaxy. What is another name of Whirlpool Galaxy? a) M31 b) M 51 c) NGC 2020 d) NGC 206 Answer: (b) Related Facts:-

 Astronomers recently found a strong possibility of planet candidate in a galaxy much far away from our Milky Way.  This galaxy is known as Whirlpool galaxy or M51  The study regarding the discovery of first new planet in a outer galaxy has been done by Rosanne Di Stefano at the Harvard-Smithsonian Center for Astrophysics along with several colleagues.  They have found a candidate planet in the M51 Whirlpool Galaxy some 23 million light years from Earth near the constellation of Ursa Major.  This new planet named as M51-ULS-1b, is probably slightly smaller than Saturn and orbits a binary star system.

32 http://www.edristi.in/

 The planet is revolving round the system at a distance of perhaps ten times Earth’s distance from the Sun.  The observation was possible because of a special set of conditions which are created by emission of X-Rays from a nearby neutron star to this system.  The X-rays emitted by this neutron star or a black hole eclipsed by this possible planet.  The X-Ray eclipse was observed by Rosanne Di Stefano and her team of researchers using the X-ray data collected by the Chandra X-Ray observatory in September 2012.  Scientists observed that source of these X-rays, the black hole or neutron star is tiny.  Scientists therefore became sure that Saturn-sized planet orbiting a billion kilometers away can completely eclipse the X-ray source, should it pass directly in front in the line of sight with Earth.  Astronomers detected that the X-ray source dimmed to nothing and then reappeared, the entire transit lasting about 3 hours.  There are various reasons why an X-ray source can dim in this way. One is the presence of another small star, such as a white dwarf, that eclipses the X-ray source.  The team says M51-ULS-1b cannot be a white dwarf or other type of star because the binary system is too young for such an object to have evolved nearby.  Scientists from all above observations concluded to the strong possibility that M51- ULS-1b must be a planet.

Link: – https://www.discovermagazine.com/the-sciences/first-evidence-of-a-planet-in-another- galaxy JIMEX-2020

Question: JIMEX-2020 is a (a) Naval Exercise (b) Air Force Exercise (c) Army Exercise (d) Coast Guard Exercise Answer: (a) Related facts:

 The 4th edition of India – Japan Maritime bilateral exercise JIMEX held in North Arabian Sea from 26 to 28 September 2020.  JIMEX is conducted biennially between the Indian Navy and Japanese Maritime Self-Defense Force (JMSDF).  JIMEX series of exercises commenced in January 2012.  The last edition of JIMEX was conducted in October 2018 off Visakhapatnam, India.

Links: https://m.economictimes.com/news/defence/4th-edition-of-india-and-japans-maritime- bilateral-exercise-jimex-2020-concludes/amp_articleshow/78380488.cms Defence Acquisition Council : Approval for various arms and equipment 33 http://www.edristi.in/

Question : Defence Acquisition Council(DAC) has recently approved for various arms and equipment worth rupees 2,290 crore.Who is the head of DAC ? (A) Defence Minister Rajnath Singh (b) Chief of the Army Staff General Manoj Mukund Naravane (c) President of India Ramnath Kovind (d) Defence Secretary Ajay Kumar Answer (a) Related facts

 The Defence Acquisition Council meeting held under the Chairmanship of Raksha Mantri Shri Rajnath today(28 September 2020) approved proposals for Capital acquisitions of various equipment required by the Indian Armed Forces at an approximate cost of Rs 2,290 crore.  These include procurement from Domestic Industry as well as Foreign vendors.  Under the Buy Indian (IDDM) category, the DAC approved procurement of Static HF Tans-receiver sets and Smart Anti Airfield Weapon (SAAW). The HF radio sets will enable seamless communication for the field units of Army and Air Force and are being procured at an approx. cost of Rs. 540 crore.  The Smart Anti Airfield Weapon being procured at an approx. cost of Rs. 970 crore will add to the fire power of Navy and Airforce.  urther, to equip the Frontline Troops of the Army the DAC also accorded approval for procurement of SIG SAUER Assault Rifles at a cost of approx. Rs.780 crore.

Link: https://pib.gov.in/PressReleasePage.aspx?PRID=1659754 Dr Reddy’s launches generic drug to treat multiple sclerosis in US

Question : Multiple sclerosis (MS) is a potentially disabling disease of the – (a) brain and spinal cord (central nervous system) (b) intestines (c) heart, arteries and veins (d) kidneys Answer (a) Related facts

 Dr Reddy’s Laboratories on Saturday(26 September 2020) said it has launched Dimethyl Fumarate delayed-release capsules, used to treat multiple sclerosis (MS), in the US market.  The company’s product is a therapeutic equivalent generic version of Biogen’s Tecfidera delayed-release capsules, the Hyderabad-based drug firm said in a statement.  According to IQVIA Health data, the Tecfidera brand and generic market had sales of around USD 3.8 billion (about Rs 28,000 crore) in the US for the most recent twelve months ending in June 2020.  Multiple sclerosis (MS) is a potentially disabling disease of the brain and spinal cord (central nervous system).

In MS, the immune system attacks the protective sheath (myelin) that covers nerve fibers and causes communication problems between our brain and the rest of our body.

34 http://www.edristi.in/

Link: https://economictimes.indiatimes.com/industry/healthcare/biotech/pharmaceuticals/dr- reddys-launches-generic-drug-to-treat-multiple-sclerosis-in- us/articleshow/78333782.cms Pixxel made an agreement to launch its second Satellite

Question: Space technology company Pixxel, which made an agreement with Momentus, to launch its second Satellite, is the company of- a) France b) UK c) USA d) India Answer: (d) Related Facts:

 On 23 September 2020; Pixxel, Bengaluru based, Indian space-tech startup, has announced about its programme to launch second satellite.  Pixxel regarding the launch of its second satellite signed an agreement with Silicon Valley-based in-space satellite transportation and infrastructure company Momentus Inc.  Pixxel’s second small satellite will be placed into the SSO (Sun-synchronous orbit) in December 2021 onboard a SpaceX Falcon-9 launch, as well as options to fly again in 2022.  The point to be noted, that Pixxel till now has not launched even its first satellite.  The company’s first satellite is scheduled to be launched towards the end of this year on a Russian Soyuz rocket.  The Momentus Shuttle Service will provide rideshare for multiple Pixxel spacecraft to predefined orbits.  Bengaluru-headquartered Pixxel is building a constellation of cutting-edge earth imaging small satellites that can provide real-time remote sensing data across the world.  Momentus launch and in-space transfer services provide this Indian Space technology start-up with the flexibility to get to the orbit it requires even on a rideshare mission.  Awais Ahmed, founder and CEO, Pixxel said that we are excited to partner with Momentus to get our satellites up in orbit as soon as we can and get the data in the hands of our customers.  About the Pixxel:-  Pixxel was founded in 2019 by then 21-year-olds, Awais Ahmed and Kshitij Khandelwal.  Pixxel satellites aim to make the world a more sustainable place through highly accurate data-based insights in agriculture, climate change and forestry.  Pixxel recently rose a funding of $5 million, which the company is using to rapidly build and launch the second satellite with Momentus’ help.  About Momentus:-  Momentus has gained significant traction since its founding in 2017  Dozens of customers since then attracted by Momentus, ranging from private commercial space to the likes of aerospace giant Lockheed Martin and US space agency NASA. 35 http://www.edristi.in/

 It has formed important industry partnerships, most notably with Elon Musk’s SpaceX.  Momentus employs new and proprietary technologies, including water plasma propulsion to enable revolutionary low-cost orbital shuttle and charter services.  The charter service will allow dedicated capacity to be allocated to a single customer’s payload.  In the Shuttle Service model, the payload will be rideshare along with other payloads hosted or deployed from the Vigoride transfer vehicle.

Links:- https://www.business-standard.com/article/companies/india-s-pixxel-partners-with- momentus-for-second-satellite-launch-120092400141_1.html Department of Biotechnology has initiated a program to strengthen clinical trial research

Question: For which of the following The Department of Biotechnology (DBT) Government of India has initiated a program to strengthen clinical trial research capacity? a) Private Hospitals b) Government Hospitals c) Neighboring countries, and LMIC d) None of the above Answer: (c) Related Facts:-

 On 22 September 2020; The Department of Biotechnology has initiated a program to strengthen clinical trial research capacity in neighboring countries.  The program will help neighboring countries, and LMIC (Low and Middle Income Country), to build capacity for COVID-19 vaccine trials.  Through this training program and knowledge sharing efforts, the Government of India is fulfilling its commitment to build various technological capabilities in its neighboring countries.  The Department of Biotechnology has initiated the first phase of the training program in partnership with the Ministry of External Affairs, Government of India.  The purpose of these trainings is to conduct clinical trial trials in compliance with ICH-GCP (International Conference on Harmonization-Good Clinical Practice).  Through these trainings, the emphasis will be on enhancing and strengthening diagnostic testing capabilities through the support of researchers and investigative teams

Link:- http://pibarchive.nic.in/newsite/erelease.aspx?relid=215508 Zero-emission commercial aircraft set to become reality Question: Airplane manufacturing company Airbus recently has revealed three concepts for the world’s first zero-emission commercial aircraft. These concepts rely on………. as primary power source. Which of the following could be correctly filled in the blank space provided

36 http://www.edristi.in/ above? a) Uranium b) Plutonium c) Hydrogen d) Phosgene Answer: (c) Related Facts:

 On 21 September 2020, Airbus has revealed three concepts for the world’s first zero-emission commercial aircraft.  These concepts each represent a different approach to achieving zero-emission flight.  All of these concepts rely on hydrogen as a primary power source.  It is expected by the company that such a concept could soon become a reality, and could enter service by 2035.  The company, exploring various technology pathways, and aerodynamic configurations, for zero-emission commercial aircraft.  Airbus want come with such technology to support the Company’s ambition of leading the way in the decarbonisation of the entire aviation industry.  Airbus believes hydrogen presents exceptional promise as a clean aviation fuel and is likely to be a solution for aerospace and many other industries to meet their climate-neutral targets.  Airbus CEO, Guillaume Faury told on the occasion that the use of hydrogen – both in synthetic fuels and as a primary power source for commercial aircraft – has the potential to significantly reduce aviation’s climate impact.”  The three concepts – all codenamed “ZEROe” – for a first climate neutral zero- emission commercial aircraft include:  A turbofan design: (120-200 passengers) with a range of 2,000+ nautical miles, capable of operating transcontinentally.  It will be powered by a modified gas-turbine engine running on hydrogen, rather than jet fuel, through combustion.  The liquid hydrogen will be stored and distributed via tanks located behind the rear pressure bulkhead.  Turboprop design: (up to 100 passengers) using a turboprop engine instead of a turbofan and also powered by hydrogen.  In it combustion of fuel will take place in modified gas-turbine engines, which  These aircraft would be capable of traveling more than 1,000 nautical miles, making it a perfect option for short-haul trips.  A Blended-Wing Body design: (capacity up to 200 passengers)  In this concept, i the wings merge with the main body of the aircraft with a range similar to that of the turbofan concept.  In it the exceptionally wide fuselage opens up multiple options for hydrogen storage and distribution, and for cabin layout.  Company said that, these concepts will help us explore and mature the design and layout of the world’s first climate-neutral, zero-emission commercial aircraft.

Link: https://www.airbus.com/newsroom/press-releases/en/2020/09/airbus-reveals-new- zeroemission-concept-aircraft.html

37 http://www.edristi.in/

IIIT Students developed ventilation device for Covid Patients

Question: Students of which of the following IIIT in India have recently developed artificial ventilation device useful for Covid patients? a) IIIT Allahabad b) IIIT Guwahati c) IIIT Kota d) IIIT Bhubaneswar Answer: (d) Related Facts:-

 Engineering students from IIIT, Bhubaneswar have developed a first-of-its-kind ventilation device that benefits patients suffering from mild respiratory distress.  It is a helmet-like device which can also enhance the treatment for Covid-19 patients with mild respiratory issues.  It can also be useful for treating respiratory diseases.  Students explained that it can also act as a comfortable alternative to the suffocating PPE kits.  It took four months to design the device named as ‘Swasner.’

Special Features:-

 SWASNER is free of maintenance, economical (can be priced at INR 1,800 per piece) and easy to use  The product is reusable after following standard disinfection procedure.  The product has got accolades and recognitions from both India and abroad.  The team had also participated in Kerala Startup Mission Hackathon.

Working Principle:

 Shaped as a bubble enclosure for the head, the device works on the principle of hyperbaric oxygen therapy.  It should be known that this therapy mostly used for treating diving-related illness and some other medical conditions.  Inside a hyperbaric chamber, a person is made to inhale almost pure oxygen to speed up body’s healing process.  Some researchers believe the therapy  In this therapy, the patient is made to breathe oxygen in the pressurised chamber. The atmospheric pressure inside the chamber is raised up to three times higher than normal.  The bubble, which expands as the air fills in, is made of medical grade plastic that these students had procured from CIPET, Bhubaneswar.  The students claimed that the device could be an affordable alternative to costly ventilators as modern ventilators may cost as high as lakh rupees.

Links:- https://indiaeducationdiary.in/iiit-bhubaneswar-students-develop-swasner-a-first-of-its- kind-helmet-like-ventilation-device/

38 http://www.edristi.in/

First Intact Planet Near a White Dwarf Discovered.

Question: An international team of astronomers found planet closely orbiting a white dwarf. Consider the following facts- I. NASA’s Transiting Exoplanet Survey Satellite (TESS) and retired Spitzer Space Telescope were used to discover it. II. The new planet is as large as Jupiter III. It has been named as WD 1856 b Which one of the statements given above is incorrect? a) (i) b) (ii) c) (iii) d) None Answer: (d) Related Facts:- • An international team of astronomers planet found closely orbiting a white dwarf. • They got it by using NASA’s Transiting Exoplanet Survey Satellite (TESS) and retired Spitzer Space Telescope. • They have reported that the planet may be the first intact planet found closely orbiting a white dwarf. • Discovery details about this planet in the Sept. 17 issue of Journal Nature. • It should be known that white dwarf is dense leftover of a Sun-like star, only 40% of the size of Earth. • This newly discovered planet called WD 1856 b, as large as Jupiter is about seven times larger than the white dwarf, named WD 1856+534. • The TESS satellite spotted WD 1856 b about 80 light-years away in the northern constellation Draco. • The planet makes round white dwarf in every 34 hours, more than 60 times faster than Mercury orbits our Sun. • It orbits a cool, quiet white dwarf, which is a distant member of a triple star system. • that is roughly 11,000 miles (18,000 kilometers) across, may be up to 10 billion years old, and • Andrew Vanderburg, an assistant professor of astronomy at the University of Wisconsin-Madison informed that the Planet WD 1856 b somehow got very close to its white dwarf and managed to stay in one piece. • It is now well known fact that the white dwarf creation process destroys nearby planets, and anything that later gets too close is usually torn apart by the star’s immense gravity. • Astronomers till now unable to explain about, how WD 1856 b arrived at its current location without annihilating in white dwarf formation process of this white dwarf.

 NASA’s Transiting Exoplanet Survey Satellite (TESS):- 39 http://www.edristi.in/

• The Transiting Exoplanet Survey Satellite (TESS) is designed to discover thousands of exoplanets in orbit around the brightest dwarf stars in the sky. • In its prime mission, a two-year survey of the solar neighborhood, TESS monitored the brightness of stars for periodic drops caused by planet transits. • The prime mission ended on July 4, 2020 and TESS is now in an extended mission. • TESS is finding planets ranging from small, rocky worlds to giant planets, showcasing the diversity of planets in the galaxy. • TESS monitors large swaths of the sky, called sectors, for nearly a month at a time. This long gaze allows the satellite to find exoplanets, or worlds beyond our solar system. • TESS do it capturing changes in stellar brightness caused when a planet crosses in front of, or transits, its star. Link:- https://www.nasa.gov/press-release/nasa-missions-spy-first-possible-survivor-planet- hugging-white-dwarf-star UNICEF to lead the supply of COVID-19 vaccines globally

Question:Consider the following with respect to COVAX Global Vaccines Facility: (1) The COVAX Global Vaccines Facility is an initiative launched in April to speed up development of medicines to treat Covid-19 and make them available to people everywhere. (2) It is operated by WHO. Which is/are correct? (a) 1 only (b) 2 only (c) Both 1 and 2 (d) None of these Answer:(a)

 Context:  In what could possibly be the world’s largest and fastest ever operation of its kind, the UNICEF will be leading the procurement and supply of coronavirus vaccines to ensure that all countries have safe, fast and equitable access to initial doses when they are available.  Facts:  The United Nations Children’s Fund (UNICEF) is the world’s largest single vaccine buyer, procuring more than 2 billion doses of various vaccines annually for routine immunisation and outbreak response on behalf of nearly 100 countries.  The UNICEF will lead efforts to procure and supply doses of Covid-19 vaccines on behalf of the COVAX Global Vaccines Facility for 92 low and lower middle- income countries, whose vaccine purchases will be supported by the mechanism.

40 http://www.edristi.in/

 The UNICEF will undertake these efforts in close collaboration with the World Health Organization (WHO), Gavi the Vaccine Alliance, the Coalition for Epidemic Preparedness Innovations (CEPI), PAHO, World Bank, the Bill and Melinda Gates Foundation, and other partners.  Additional Fact:  COVAX Global Vaccines Facility?  The COVAX Global Vaccines Facility is the vaccine pillar of the ACT-Accelerator, an initiative launched in April to speed up development of medicines to treat Covid-19 and make them available to people everywhere.  It is operated by Gavi, the Vaccine Alliance; CEPI, and WHO, alongside multinational and developing country vaccine manufacturers.

Link: https://www.thehindu.com/news/international/unicef-to-lead-global-procurement-supply- of-covid-vaccines/article32539511.ece World Solar Technology Summit

Question:Consider the following with respect to first World Solar Technology Summit: (1) It has been organised by International Solar Alliance(ISA) and Federation of Indian Chambers of Commerce & Industry(FICCI). (2) Two PSUs under the Ministry of Petroleum & Natural Gas will be joining International Solar Alliance Coalition for Sustainable Climate Action. Which is/are correct? (a) 1 only (b) 2 only (c) Both 1 and 2 (d) None of these Answer:(a)

 Context:  Union Minister of Petroleum and Natural Gas has addressed the first World Solar Technology Summit.  Facts:  World Solar Technology Summit(WSTS): It has been organised by International Solar Alliance(ISA) and Federation of Indian Chambers of Commerce & Industry(FICCI).  Aim: To bring together key stakeholders to discuss the highlights of solar technologies, cost-wise; technology-wise, technology transfers, challenges and concerns in the field.  Key Takeaways:  PSU’s: Five Public Sector Undertakings(PSUs) under the Ministry of Petroleum & Natural Gas will be joining International Solar Alliance (ISA)’s Coalition for Sustainable Climate Action(ISA-CSCA) as Corporate Partners.  Project Preparation Facility: It has been set up to develop bankable Solar Energy projects in ISA member countries with the help of EXIM Bank of India.  Additional Facts:

41 http://www.edristi.in/

 International Solar Alliance(ISA): It is a treaty-based international intergovernmental organization jointly launched by India and France on the side-lines of the COP 21 to the United Nations Framework Convention on Climate Change in 2015.  Aim: To work for efficient exploitation of solar energy to reduce dependence on fossil fuels.  Objectives:  To collectively address key common challenges to scale up solar energy applications in line with their needs  To mobilize investments of more than USD 1000 billion by 2030;  To bring about a major decrease in the cost of solar energy  To scale up applications of solar technologies in member countries and facilitate collaborative research and development (R&D).  Members: ISA Framework Agreement has been signed by 86 countries, with 68 having also deposited instruments of ratification.  Headquarters: Gurugram, India.

Link: https://energy.economictimes.indiatimes.com/news/renewable/world-solar-technology- summit-to-see-26k-participants-from-149- nations/77988853#:~:text=M%20Stanley%20Whittingham%2C%20winner%20of,will%2 0also%20address%20the%20inaugural. Hypersonic Test Demonstration Vehicle Question:Hypersonic Test Demonstration Vehicle has been developed by: (a) Hindustan Aeronauticals ltd (b) ISRO (c) DRDO (d) IISc Bangalore Answer:(c)

 Context:  Prime Minister Narendra Modi has congratulated the Defence Research and Development Organisation (DRDO) for successful flight of the Hypersonic Test Demonstration Vehicle (HSTDV).  Facts:  HSTDV: It is an unmanned scramjet demonstration aircraft for hypersonic speed flight.  Purpose: It is being developed as a carrier vehicle for hypersonic and long-range cruise missiles and will have multiple civilian applications including missiles of the future for air defence, surveillance besides in the development of energy-efficient, low cost and reusable satellite-launch vehicles.  Developed by: It has been developed by the Indian Defence Research and Development Organisation(DRDO).  Additional Fact:  Ramjet:  It is a form of air-breathing jet engine that uses the vehicle’s forward motion to compress incoming air for combustion without a rotating compressor.

42 http://www.edristi.in/

 Fuel is injected in the combustion chamber where it mixes with the hot compressed air and ignites.A ramjet-powered vehicle requires an assisted take-off like a rocket assist to accelerate it to a speed where it begins to produce thrust.  Ramjets work most efficiently at supersonic speeds around Mach 3 (three times the speed of sound) and can operate up to speeds of Mach 6.However, the ramjet efficiency starts to drop when the vehicle reaches hypersonic speeds.  Scramjet:  It is an improvement over the ramjet engine as it efficiently operates at hypersonic speeds and allows supersonic combustion.Thus it is known as Supersonic Combustion Ramjet or Scramjet.

Link: https://www.pib.gov.in/PressReleasePage.aspx?PRID=1651956 National Immunogenicity & Biologics Evaluation Center

Question:Consider the following with respect to National Immunogenicity & Biologics Evaluation Center: (1) It is a first of its kind centre in the country developed for evaluation of the immunogenicity of vaccines & antiviral properties of drugs/preparations. (2) It has been established jointly by Bharati Vidyapeeth University and BIRAC-DBT. Which is/are correct? (a) 1 only (b) 2 only (c) Both 1 and 2 (d) None of these Answer:(c)

 Context:  Department of Biotechnology(DBT) Secretary has inaugurated National Immunogenicity & Biologics Evaluation Center(NIBEC).  Facts:  About NIBEC: It is a first of its kind centre in the country developed for evaluation of the immunogenicity of vaccines & antiviral properties of drugs/preparations.  Developed by: It has been established jointly by Bharati Vidyapeeth University through its constituent unit Interactive Research School for Health Affairs(IRSHA) and BIRAC-DBT through National Biopharma Mission.  Additional Facts:  National Biopharma Mission(NBM):  It is an industry-academia collaborative mission for accelerating biopharmaceutical development in the country.  Launched Year: It was launched in 2017 at a total cost of Rs 1500 crore and is 50% co-funded by World Bank loan.  Implementation: Biotechnology Industry Research Assistance Council (BIRAC).  Innovate in India(i3) programme: It has been launched under this mission to create an enabling ecosystem to promote entrepreneurship and indigenous manufacturing in the biopharma sector.  BIRAC:

43 http://www.edristi.in/

 It is a not-for-profit state owned enterprise under the Department of Biotechnology (DBT) to strategically empower emerging biotech companies.

Link: https://pib.gov.in/PressReleasePage.aspx?PRID=1651639 Chandrayaan-3

Question:Consider the following with respect to Chandrayaan-3: (1) It will be a mission repeat of Chandrayaan-2 (2) It will include a Lander and Rover similar to that of Chandrayaan-2, but will not have an orbiter. Which is/are incorrect? (a) 1 only (b) 2 only (c) Both 1 and 2 (d) None of these Answer:(d)

 Context:  Minister of State for Space Dr Jitendra Singh has said that the launch of Chandrayaan-3 may now take place somewhere in early 2021.  Facts:  Chandrayaan-3 will be a mission repeat of Chandrayaan-2 and will include a Lander and Rover similar to that of Chandrayaan-2, but will not have an orbiter.  He also said that Images sent by Chandrayaan-1 show that Moon may be rusting along the poles. The sign of this finding is that even though the surface of the Moon is known to have iron – rich rocks, it is not known for the presence of water and oxygen, which are the two elements needed to interact with iron to create rust.

Link: http://newsonair.com/Main-News-Details.aspx?id=399241 Chandrayaan-3

Question:Consider the following with respect to Chandrayaan-3: (1) It will be a mission repeat of Chandrayaan-2 (2) It will include a Lander and Rover similar to that of Chandrayaan-2, but will not have an orbiter. Which is/are incorrect? (a) 1 only (b) 2 only (c) Both 1 and 2 (d) None of these Answer:(d)

 Context:  Minister of State for Space Dr Jitendra Singh has said that the launch of Chandrayaan-3 may now take place somewhere in early 2021.

44 http://www.edristi.in/

 Facts:  Chandrayaan-3 will be a mission repeat of Chandrayaan-2 and will include a Lander and Rover similar to that of Chandrayaan-2, but will not have an orbiter.  He also said that Images sent by Chandrayaan-1 show that Moon may be rusting along the poles. The sign of this finding is that even though the surface of the Moon is known to have iron – rich rocks, it is not known for the presence of water and oxygen, which are the two elements needed to interact with iron to create rust.

Link:

http://newsonair.com/Main-News-Details.aspx?id=399241 Pinaka missile system

Question:Consider the following with respect to Pinaka missile system: (1) It is a multiple rocket launcher produced in India and developed by the ISRO for the Indian Army. (2) Pinaka can be used for attacking the adversary targets prior to the close quarter battles which involve smaller range artillery, armoured elements and the infantry. Which is/are correct? (a) 1 only (b) 2 only (c) Both 1 and 2 (d) None of these Answer:(b)

 Context:  The Ministry of Defence(MoD) has announced that it’s acquisition wing had signed contracts with three Indian companies for supply of six regiments of the Pinaka Rocket System to be deployed along borders with Pakistan and China.  Facts:  Pinaka: It is a multiple rocket launcher produced in India and developed by the Defence Research and Development Organisation for the Indian Army.  Range: The system has a maximum range of 40 km for Mark-I and 75 km for Mark- II, and can fire a salvo of 12 HE rockets in 44 seconds.  Significance: Pinaka can be used for attacking the adversary targets prior to the close quarter battles which involve smaller range artillery, armoured elements and the infantry.

Link: https://indianexpress.com/article/explained/pinaka-rocket-system-acquisition-features- capabilities-origin- 6578723/#:~:text=The%20acquisition%20wing%20of%20MoD,cost%20of%20Rs%202 %2C580%20crore. India Russia biennial naval exercises

Question:The 11th edition of Exercise INDRA NAVY recently held,it is a biennial bilateral maritime exercise between India and : (a) Japan 45 http://www.edristi.in/

(b) Brazil (c) Russia (d) Australia Answer:(c)

 Context:  The 11th edition of exercise INDRA NAVY, a biennial bilateral maritime exercise between Indian Navy and Russian Navy is scheduled in the Bay of Bengal from 04 to 05 September 2020.  Facts:  Initiated in 2003, Ex INDRA NAVY epitomises the long-term strategic relationship between the Navies of India and Russia.  The primary aim of exercise INDRA NAVY-20 is to further consolidate inter- operability built up by the two Navies over the years and also to enhance understanding and procedures for multi-faceted maritime operations.  The Indian Navy will be represented by guided missile destroyer Ranvijay, indigenous frigate Sahyadri and fleet tanker Shakti, along with their integral helicopters.  The last edition of the exercise was conducted off Visakhapatnam in December 2018.

Link: https://www.timesnownews.com/india/article/indra-navy-2020-indian-russian-navies- begin-naval-exercise-guided-missile-destroyers-frigates-in- action/647623#:~:text=New%20Delhi%3A%20The%2011th%20edition,%2C%20at%20s ea%20only’%20format. Aatmanirbhar Bharat’

Question:Giving a push to the ‘Aatmanirbhar Bharat’ call by the Prime Minister, who has launched an alternative to Camscanner, AIR SCANNER APP: (a) IIT Kharagpur (b) IIT Bombay (c) IIT Madras (d) IIT Delhi Answer:(b)

 Context:  Giving a push to the ‘Aatmanirbhar Bharat’ call by the Prime Minister, IIT Bombay has launched an alternative to Camscanner, a Chinese app mainly used for scanning documents, that was banned by the Government of India.  Facts:  The new app launched by IIT Bombay students is called as AIR SCANNER and is available free of cost.  IIT Bombay has developed an indigenous alternative to Chinese document-scanning app, following the Centre’s decision to ban 59 mobile apps.

Link:

46 http://www.edristi.in/ http://newsonair.com/News?title=IIT-Bombay-students-develop-AIR-Scanner%2C-an- Indian-alternative-to- Camscanner&id=398785#:~:text=Giving%20a%20push%20to%20the,is%20available% 20free%20of%20cost. Health assistant Robot Rakshak designed by Railways Question:Who among the following has designed a health assistant Robot Rakshak which can remotely communicate between doctor and patient: (a) ISRO (b) DRDO (c) Indian Railways (d) IIT Madras Answer : (c)

 Context:  In the fight against the spread of Covid 19 pandemic, the Railways has designed a health assistant Robot Rakshak which can remotely communicate between doctor and patient.  Facts:  The medical aid robot is able to measure health parameters such as temperature, pulse, oxygen percentage.  It can also provide medicines, food to the patients and make a two-way video communication between the doctor and the patient.  It can move in all directions at all levels with a range of remote operations up to 150 meters.  With a full charge battery, the Rakshak robot can work continuously for 6 hours and can carry up to 10 kg of weight in its tray. It is based on Wi-Fi and therefore does not require any mobile data. It also operates with an Android mobile application.

Link: http://www.newsonair.com/News?title=Health-assistant-Robot-Rakshak-designed-by- Railways-can-remotely-communicate-between-doctor-and- patient&id=398769#:~:text=In%20the%20fight%20against%20the,temperature%2C%20 pulse%2C%20oxygen%20percentage. World’s Largest Solar Tree

Question:Consider the following with respect to Solar Tree: (1) It is a structure incorporating solar energy technology on a single pillar, like a tree trunk. (2) World’s Largest Solar Tree is installed at CSIR-CMERI Durgapur, West Bengal. Which is/are correct? (a) 1 only (b) 2 only (c) Both 1 and 2 (d) None of these Answer:(c)

 Context: 47 http://www.edristi.in/

 CSIR-CMERI has developed the World’s Largest Solar Tree which is installed at CSIR-CMERI Durgapur, West Bengal.The installed capacity of the Solar Tree is above 11.5 kWp.The Solar tree has an annual capacity of generating 12,000 to 14,000 units of Clean and Green power.  Facts:  Solar Tree can be aligned with agriculture to substitute price volatile fossil fuels.One solar tree can reduce ten to twelve tonnes of carbon dioxide emissions.Also,the surplus power from these trees can be fed to the energy grids.  It also has capability to incorporate Internet of Things(IoT) based features.This includes wind speed, real-time humidity, rainfall prediction and soil analytics sensors.  Additional Facts:  e-Suvidha: These are solar powered kiosks developed by CSIR-CMERI that can be connected to Solar Trees for real-time access to the vast majority of agricultural database as well as to the eNAM i.e. National Agricultural MarketPlace for instant and real-time access to an unified online market.  Solar Tree: It is a structure incorporating solar energy technology on a single pillar, like a tree trunk.It may be a solar artwork or a functional power generator.

Link: http://ddnews.gov.in/sci-tech/csir-cmeri-develops-world%E2%80%99s-largest-solar- tree#:~:text=According%20to%20Dr%20Harish%20Hirani,wp%20(watts%20peak)%20e ach. KAVKAZ 2020

QuestionConsider the following statements with respect to KAVKAZ 2020: (1) Both China and Pakistan are going to be a part of the multinational exercise to be held in Russia. (2) India has announced it’s withdrawl from this multilateral Military Exercise. Which is/are incorrect? (a) 1 only (b) 2 only (c) Both 1 and 2 (d) None of these Answer : (d) Context:

 India has announced that it is withdrawing from multilateral Military Exercise Kavkaz 2020 to be held in Russia.  Facts:  India had earlier planned to send around 150 Indian Army troops, 45 Indian Air Force personnel and a number of Navy officers to the multilateral military exercise to be held in the Astrakhan region in southern Russia from 15th to 26th September.  Both China and Pakistan are going to be a part of the multinational exercise. China’s participation in the exercise was also a reason for India to pull out of the multilateral tri-services military exercise.  India is locked in a military conflict with the Chinese in eastern Ladakh and on high alert all along the 4,000 kilometre Line of Actual Control (LAC). 48 http://www.edristi.in/

Link: http://www.newsonair.com/News?title=India-withdraws-from-multilateral-military- exercise-in-Russia-next-month&id=398571 IIT-Indore to teach ancient Indian sciences in Sanskrit Question:Which IIT has introduced a one of its kind course to teach classical scientific texts in Sanskrit? (a) Delhi (b) Kanpur (c) Indore (d) Patna Answer:(c) Context:

 IIT-Indore has introduced a one-of-its-kind course to teach classical scientific texts to students in Sanskrit, starting with Bhaskaracharya’s mathematical treatise ‘Lilavati’ of almost a thousand years ago.  Facts:  More than 750 people from across the globe have signed up for the course that started on August 22. The first edition will end on October 2, Gandhi Jayanti.  The institute has lined up fortnightly programmes to teach metallurgy, astronomy, medicines and plant sciences in Sanskrit. Students can study classical Indian scientific texts in their original forms and converse about them in the ancient language.  Most of the traditional Indic scientific treatises in sustainable water resource management, agriculture, mathematics, metallurgy, astronomy, medicine, plant sciences, economic and political texts are in Sanskrit. Understanding Sanskrit with a view to studying these texts is important to preserve India’s scientific heritage.

Link: https://timesofindia.indiatimes.com/india/iit-indore-to-teach-ancient-indian-science-in- sanskrit/articleshow/77813378.cms

Sports

Tennis Italian Open 2020 Men’s Singles

Question:Who has won the Italian Open 2020 Men’s Singles ? (a) Novak Djokovic (b) Diego Schwartzman (c) Rafael Nadal (d) Roger Federer Answer: (a) Related facts: 49 http://www.edristi.in/

 In tennis, world number one Novak Djokovic won Italian Open at Rome on September 21 2020.  He defeated Diego Schwartzman by 7-5, 6-3 in the final.  With this victory, Djokovic Masters title overall count rose to 36.

Links: http://newsonair.com/News?title=Novak-Djokovic-wins-Men%26%2339%3bs-Singles- event–in-Italian-Open&id=400226 2020 Italian Open – Women’s Singles Question:Who has won the 2020 Italian Open – Women’s Singles? (a) Simona Halep (b) Karolína Plíšková (c) Sofia Kenin (d) Elina Svitolina Answer: (a) Related facts:

 Top-seeded Simona Halep of Romania won her first Italian Open title.  She has been declared winner after defending champion Czech Karolína Pliskova retired midway through the final with an injury.  Halep was leading 6-0, 2-1 when Czech player Pliskova stopped playing after just 31 minutes.

Links: http://newsonair.com/Main-News-Details.aspx?id=400327

Sports Personality Koneru Humpy wins Armageddon

Question:Armageddon is related to which of the following sports? (a) Rugby (b) Chess (c) Hockey (d) Football Answer:(b) Context:

 Koneru Humpy won in an Armageddon against Poland’s Monica Socko to take India into final of the first-ever online Chess Olympiad. It has been dubbed as one of the most exciting Armageddon shootouts of recent times.  Facts:  ‘Armageddon’ in chess is similar to a Super Over in cricket or a penalty shootout in hockey or football. 50 http://www.edristi.in/

 In an Armageddon game White has more thinking time on the clock than Black but a draw on the board scores as a Black win. Normally White has five or six minutes and Black four or five for the entire game.  Black, considered at a disadvantage, is given a minute less to process the game and the handicap effectively puts the onus of a win on White.

Link: https://www.indiatoday.in/sports/other-sports/story/koneru-humpy-helps-india-beat- poland-to-reach-online-chess-olympiad-final-1716572-2020-08-29

Short Notes

Personalities D K Kashyap

Question: RBI has appointed D K Kashyap as director on board of—- (a) Dhanlaxmi Bank (b) Karur vaishya bank (c) HDFC bank (d) CSB Answer- a Related facts

 RBI has recently (September 2020) appointed D K Kashyap as director on board of Dhanlaxmi Bank.  He is the current GM of the RBI.  RBI has appointed D K Kashyap on the board of the bank for two years.  The appointment is for a period of two years with effect from September 28, 2020, to September 27, 2022, or till further orders, whichever is earlier  The RBI usually does not appoint its nominee on the boards of private banks unless there are exceptional circumstances to avoid any conflict of interest.  Kerala-based Dhanlaxmi Bank was put under the Prompt Corrective Action (PCA) Framework by the RBI in November 2015 due to deteriorating financial health and only last year it came out of these restrictions. Since then it has posted profit.  Another private lender Lakshmi Vilas Bank (LVB), which was put under the PCA framework in September 2019, has also come under the RBI’s watch.  The RBI on Sunday(27September 2020) approved appointment of a three-member Committee of Directors (CoD) to run the daily affairs of Lakshmi Vilas Bank after bank shareholders ousted seven directors of the debt-ridden lender.  This CoD will exercise the discretionary powers of MD & CEO in the ad-interim.

Link: https://m.economictimes.com/industry/banking/finance/banking/rbi-appoints-d-k- kashyap-as-director-on-board-of-dhanlaxmi-bank/articleshow/78389299.cms TRAI chairman

51 http://www.edristi.in/

Question:Who has been appointed the Chairman of Telecom Regulatory Authority of India- TRAI? (a) Dr P. D. Vaghela (b) P S Sringala (c) Anil Dhasmana (d) Rajeev Kumar Answer: (a) Related facts:

 Senior Indian Administrative Service Officer Dr P. D. Vaghela has been appointed as the Chairman of Telecom Regulatory Authority of India- TRAI.  He will replace R.S. Sharma.  Mr Vaghela will hold the post for a period of three years or until he attains the age of 65 years.  Mr Vaghela, a 1986-batch officer of Gujarat Cadre, is currently Secretary, Department of Pharmaceuticals.

Links: http://newsonair.com/News?title=Senior-IAS-PD-Vaghela-appointed-as-new-TRAI- chairman&id=400971 SP Balasubrahmanyam

Question: Consider the following statements: (1) SP Balasubrahmanyam won six National Film Awards for Best Male Playback Singer. (2) He was conferred Padma Bhushan in 2011. Of the above correct statement/s is/are: (a) Only 1 (b) Only 2 (c) Both 1&2 (d) None of the above Answer: (c) Related facts:

 On 25th September 2020, Iconic Indian singer SP Balasubrahmanyam died in Chennai, Tamil Nadu, after suffering a cardiorespiratory arrest.He was 74.  In August he was hospitalized following Covid 19 illness. The singer tested negative almost a month later on September 4.  In his 50-year career,S P (Sripathi Panditaradhyula) Balasubrahmanyam sang over 40,000 songs, a Guinness World Record, in 16 languages, predominantly in Telugu, Tamil, Kannada, Hindi and films.  He won six National Film Awards for Best Male Playback Singer for his works in four different languages; Kannada, Telugu, Tamil and Hindi.  In 2012, he received the state NTR National Award for his contributions to Indian cinema.  He was conferred Padma Shri (2001) and Padma Bhushan (2011) from the Government of India.

52 http://www.edristi.in/

Links: https://www.aljazeera.com/news/2020/9/25/legendary-indian-singer-sp- balasubramanyam-passes-away https://en.wikipedia.org/wiki/S._P._Balasubrahmanyam Jaswant Singh

Question:Jaswant Singh who died on September 27,2020 was a- (a) Hockey Player (b) Politician (c) Wrestler (d) Musician Answer: (b) Related facts:

 Former Union Minister and BJP leader Jaswant Singh passed away on September 27,2020.  Major Jaswant Singh Jasol (1938 –2020) was one of the founding members of the Bharatiya Janata Party (BJP).  During the Vajpayee administration between 1998 and 2004, he held multiple cabinet portfolios including Finance, External Affairs and Defence.  He also served as the Deputy Chairman of the Planning Commission between 1998 and 1999.  He served as Leader of Opposition in the Rajya Sabha from 2004 to 2009.  In 2009, he was expelled from BJP accused of praising Muhammad Ali Jinnah, in his book, Jinnah – India, Partition, Independence.

Links: https://en.wikipedia.org/wiki/Jaswant_Singh Isher Judge Ahluwalia

Question: Renowned personality Isher Judge Ahluwalia was (a) Politician (b) Economist (c) Business tycoon (d) Cartoonist Answer :(b) Related facts:

 Renowned economist Isher Judge Ahluwalia passed away on September 26,2020.  The 74-year old died battling brain cancer over the past ten months.  The wife of former Deputy Chairman of the Planning Commission Montek Singh Ahluwalia, Isher served as the head of the Indian Council for Research on International Economic Affairs for 15 years.  She stepped down from the post on August 10 due to poor health.  She was awarded India’s third highest civilian award, Padma Bhushan in 2009.

Links: https://www.doordarshannews.com/eminent-economist-isher-judge-ahluwalia-dead/

53 http://www.edristi.in/

Rahul Parikh

Question : Which Bank has announced Rahul Parikh as its digital head to spearhead digital banking business ? (a) Bandhan Bank (B) Axis Bank (c) HDFC Bank (d) ICICI Bank Answer (a) Related facts

 Private sector Bandhan Bank has recently(September 2020) announced the appointment of Rahul Parikh as digital head as the bank broadens its digital presence to improve customer services.  Especially in banking, customers are adopting digital services at a very high rate, which has accelerated in the last few months.  As the chief marketing & digital officer, Parikh will be responsible for the digital banking business and for building a strong digital backbone to support the bank’s growth.  The bank’s digital banking head Rustam Nongmaithem will report him.  Parikh joined Bandhan from Roots Ventures, where he was a partner and responsible for fundraising, supporting deal-making and investee company operations. He was earlier chief executive officer with Bajaj Capital.  Over the next five years, India is set to experience the biggest boom in digital- banking adoption, with a 21% increase in the number of adults with online-only bank accounts.  This means that by 2025, we estimate that just under 400 million Indian adults will hold neobank accounts.

Link: https://economictimes.indiatimes.com/industry/banking/finance/banking/bandhan-bank- announces-rahul-parikh-as-its-digital-head-to-spearhead-digital-banking- business/articleshow/78378129.cms Chief of NTRO

Question: Who has been appointed as new chief of National Technical Research Organisation (NTRO)? (a) Samant Goel (b) Arvind Kumar (c) Anil Dhamsana (d) Rajinder Khanna Answer: (c) Related facts:

 Former Research and Analysis Wing (R&AW) chief Anil Dhasmana was appointed as the new chief of National Technical Research Organisation (NTRO).  Dhasmana would be the head of NTRO for next two years.  He will replace former Intelligence Bureau (IB) officer Satish Jha.

54 http://www.edristi.in/

Links: https://www.indiatoday.in/india/story/former-raw-chief-anil-dhasmana-appointed-as-new- chief-of-national-technical-research-organisation-1723242-2020-09-19 New Prime Minister of Japan

Question: Who took over as the new Prime Minister of Japan on September 16, 2020? (a) Yoshihide Suga (b) Taro Aso (c) Shinzo Abe (d) Fumio Kishida Answer: (a) Related facts:

 On 16 September 2020, Yoshihide Suga took over as the new Prime Minister of Japan.  He succeeded Shinzo Abe.  Earlier he was the Chief Cabinet Secretary of the country.  Recently he became the president of Japan’s ruling Liberal Democratic Party (LDP).

Links: https://www.bbc.com/news/world-asia-54172461 Neelakanta Bhanuprakash

Question:Neelakanta Bhanuprakash is famous for: (a) 1st Indian to bought land on Mars. (b) New chairman of DRDO. (c) The Fastest Human Calculator of the World. (d) None of these Answer:(c)

 Context:  The Fastest Human Calculator of the World, Neelakanta Bhanuprakash and an aspiring educational activist, is advocating for change in the way Mathematics is being taught in schools currently.  Facts:  Speaking about his concept called ‘Vision Math’ Mr Bhanuprakash said the many European countries amongst others, have expressed keen interest in it.  The 20 year old maths graduate, won the first ever Gold Medal for India in the Mental Calculation World Championship at Mind Sport Olympiad in London recently.  He aspires to produce Math Geniuses from India especially from Government schools with an aim to set up the World’s 1st Brain Training Lab in Telangana.

Link: http://www.newsonair.com/News?title=Bhanuprakash-advocates-for-change-in-way- Mathematics-being-taught-in-schools-currently&id=399278 55 http://www.edristi.in/

Question:Consider the following statements with respect to Kesavananda Bharati case : (1) The case was primarily about the extent of Parliament’s power to amend the Constitution. (2) This case gives birth to the legal doctrine of Basic Structure. Which is/are correct? (a) 1 only (b) 2 only c) Both 1 and 2 (d) None of these Answer:(c)

 Context:  Prime Minister Narendra Modi expressed grief over the passing away of Kesavananda Bharati Ji.  Facts:  Kesavananda Bharati was the head seer of the Mutt in of Kerala since 1961. He left his signature in one of the significant rulings of the Supreme Court when he challenged the Kerala land reforms legislation in 1970.  The Kesavananda Bharati judgement, is a landmark decision of the Supreme Court of India that outlined the basic structure doctrine of the Constitution.  Justice Hans Raj Khanna asserted through the Basic Structure doctrine that the constitution possesses a basic structure of constitutional principles and values.  The doctrine forms the basis of power of the Indian judiciary to review and override amendments to the Constitution of India enacted by the Indian parliament.  What was the Kesavananda Bharati case about?  The case was primarily about the extent of Parliament’s power to amend the Constitution.  The case initially started on the challenge to the Kerala Land Reforms (Amendment) Act 1969.  Later, the court was reviewing a 1967 decision in Golaknath vs State of Punjab which had reversed the earlier verdicts and ruled that Parliament cannot amend fundamental rights.  The court was also deciding the constitutional validity of several amendments passed by the Government.  What was the judgement?  The Constitutional Bench ruled that Parliament could amend any part of the Constitution so long as it did not alter or amend the basic structure or essential features of the Constitution.  However, the court did not define the ‘basic structure’ and has been adding new features to this concept through various SC judgments which includes: supremacy of the Constitution, rule of law, Independence of the judiciary, doctrine of separation of powers, federalism, secularism, principle of free and fair elections among others.

Link: https://pib.gov.in/PressReleaseIframePage.aspx?PRID=1651787 William Shatner Question:Consider the following : (1)Agender: Someone who identifies as not belonging to any gender. (2)Androgynous: Someone who identifies as neither man nor woman.

56 http://www.edristi.in/

(3)Bigender: Someone who identifies as both man and woman. Which is/are correct? (a) 1 and 2 (b) 3 only (c) 2 and 3 (d) 1,2 and 3 Answer:(d) Context:

 Last month, Hollywood actor William Shatner took offence to being called ‘CIS’, short for ‘cisgendered’.  Facts:  The term cisgendered is used to define people whose gender identity matches the identity assigned to them at birth.  For transgender people, their sense of gender identity does not match the one assigned to them at birth.  The latin prefix ‘cis’ literally means ‘on the same side of’, while ‘trans’ means on the other side.  Why the word is important? Quite simply, if there are ‘transgendered’ people, there should be a word for those who are not. Having distinct words for transgendered and cisgendered people denotes that both are equally valid, neutral experiences, with neither being an aberration.  Additional Fact:  Some identity labels in use?  Agender: Someone who identifies as not belonging to any gender.  Androgynous: Someone who identifies as neither man nor woman.  Bigender: Someone who identifies as both man and woman.  Non-binary: Someone who rejects the binaries of male and female.  Genderfluid: Someone whose gender identity changes.  Genderquestioning: Someone who is exploring which gender they identify as.  Genderqueer: An umbrella term for people not subscribing to traditional genders.  AFAB, AMAB: Assigned Female At Birth, Assigned Male At Birth.  Intersex: Those who do not possess the physical characteristics of either males or females.  Third Gender: Those who have a gender identity beyond man or woman.

Link: https://indianexpress.com/article/explained/cisgender-agender-bigender-genderqueer- what-are-these-terms-why-do-they-matter-6581592/ Dinesh Kumar Khara

Question:The Banks Board Bureau recommended Dinesh Kumar Khara for the post of chairman of ? (a) PNB (b) SBI (c) ICICI (d) HDFC Answer:(b) Context:

 The Banks Board Bureau recommended Dinesh Kumar Khara for the post of chairman of the State Bank of India (SBI).  Facts: 57 http://www.edristi.in/

 Constituted in 2016, Banks Board Bureau makes recommendations for the appointment of whole-time directors as well as non-executive chairpersons of public sector banks (PSBs).  The present SBI chairman incumbent Rajnish Kumar will demit office on October 7. Kumar was appointed for a period of three years from October 2017.

Link: https://www.business-standard.com/article/finance/banks-board-bureau-recommends- dinesh-khara-for-sbi-chairman-post-120082801606_1.html Pranab Mukherjee Question: Consider the following statements regarding Former President Pranab Mukherjee : (i) He was the 12th President of India from the year 2013 to 2017. (ii) He was appointed deputy chairman of the Planning Commission in 1991. (iii) He was awarded the Bharat Ratna, the highest civilian honor of India in year 2019. Of the above correct statements are: (a) Only (i), (ii) and (iii) (b) only (ii) and (iii) (c) only (i) and (iii) (d) All the above Answer: (b) Related facts:

 Former Indian President Pranab Mukherjee passed away on August 31,2020.He was 84.  Pranab Kumar Mukherjee (1935 – 2020) served as the 13th President of India from 2012 until 2017.  He was a senior leader in the Indian National Congress and occupied several ministerial portfolios in the Government of India.  He first served as Finance Minister of India in 1982–84. He was also the Leader of the House in the Rajya Sabha from 1980 to 1985.  He was appointed deputy chairman of Planning Commission in 1991 and Foreign Minister in 1995 in P.V. Narasimha Rao government.  From 2004 until his resignation in 2012, Mukherjee held a number of key cabinet portfolios in Prime Minister Manmohan Singh’s government – Defence (2004–06), External Affairs (2006–09) and Finance (2009–12) – apart from being Leader of the House in the Lok Sabha.  He was awarded India’s highest civilian honour, the Bharat Ratna in 2019.

Links: https://en.wikipedia.org/wiki/Pranab_Mukherjee

Places Official Visit of External Affairs Minister to Finland

Question:Recently, External Affairs Minister S. Jaishankar was on state visit to which European Union country? (a) Finland (b) France (c) Sweden (d) Germany

58 http://www.edristi.in/

Answer:(a) Related facts:

 Dr. S. Jaishankar, External Affairs Minister (EAM) was on an official visit to Finland from 19-21 September, 2019.  He was received by Finnish President Sauli Niinistö.  S Jaishankar held talks with Prime Minister Antti Rinne and his Finnish counterpart Pekka Haavisto on a broad spectrum of bilateral relations.  During his state visit, Minister Jaishankar and his Finnish counterpart Pekka Haavisto unveiled the statue of Mahatma Gandhi in Helsinki as a part of commemoration of 150th birth anniversary of Mahatma Gandhi.  He also met the Indian Community there and underlined the Mahatma’s message of truth,courage and hope.  External Affairs Minister held the long discussions on cross-border terrorism emphasizing Pakistan as the “epicentre of global terrorism”amidst Pakistan’s bid to internationalise the Kashmir issue after India revoked Jammu and Kashmir’s special status.Finland has supported India on the Kashmir issue.  The three-day visit discussed countries’ bilateral relations with special focus on green technologies.  Jaishankar also visited the Parliament of Finland and held talks with first deputy speaker Tuula Haatainen.  Finland, new President of European Union Council,is a Northern European nation bordering Norway, Russia and Sweden by land and Estonia by sea.  India’s trade in goods with Finland crossed $ One billion and is in Finland’s favour. Finland is India’s 60th largest trade partner globally, and the 10th largest within the EU.

Links: https://www.thehindu.com/news/international/jaishankar-holds-talks-on- terrorism-regional-issues-with-top-finnish-leadership/article29475681.ece Feroz Shah Kotla Stadium to be renamed as Arun Jaitley Stadium

Question: Firoz Shah Kotla Stadium is to be renamed on which political personality? (a) Arun Jaitley (b) Rajiv Gandhi (c) Venkaiah Naidu (d) Atal Bihari Vajpayee Answer :(a) Related facts:

 Feroz Shah Kotla Stadium in Delhi will be renamed as Arun Jaitley Stadium in honour of former DDCA President.  The stadium will be renamed on September 12 at a ceremony.  The DDCA decided to rename the stadium after the former DDCA president, Arun Jaitley, who breathed his last on August 24, 2019.

Links: https://www.news18.com/cricketnext/news/feroz-shah-kotla-to-be-renamed-arun-jaitley- stadium-2286121.html

59 http://www.edristi.in/

Awards & Honors 55th Jnanpith Award

Question: Who won 55th Jnanpith Award? (a) AkkithamAchuthan Namboothiri (b) Amitav Ghosh (c) Krishna Sobti (d) Shankha Ghosh Answer: (a) Related facts:

 On 24th September 2020, renowned Malayalam poet AkkithamAchuthan Namboothiri was conferred the 55th Jnanpith Award.  Akkitham became the sixth writer to bring Jnanpith to Malayalam literature.  Jnanpith Award given by Bharatiya Jnanpith, is conferred annually to an author for their “outstanding contribution towards literature”.  It is bestowed only on Indian writers writing in Indian languages included in the Eighth Schedule to the Constitution of India and English.  The award carries cash prize is ₹11 lakh and a Bronze replica of Goddess Saraswati.

Links: https://www.thehindu.com/news/national/kerala/jnanpith-given-to- akkitham/article32685581.ece https://en.wikipedia.org/wiki/Jnanpith_Award Indira Gandhi Peace Prize for 2019

Question:Who was conferred with the Indira Gandhi Peace Prize for 2019 ? (a) Centre for Science and Environment (b) Dr Manmohan Singh (c) Sir David Attenborough (d) United Nations High Commissioner for Refugees Answer:(c)

 Context:  Former Prime Minister Manmohan Singh conferred the Indira Gandhi Peace Prize for 2019 on British broadcaster Sir David Attenborough, whom he described as the “human voice of nature”. Sir David, 94, is well known for his series The Living Planet.  Facts:  The award is conferred by Indira Gandhi Memorial Trust each year on the late Prime Minister’s birth anniversary.  The award is given to individuals or organisations in recognition of creative efforts toward promoting international peace, development, ensuring that scientific discoveries are used for the larger good of humanity, and enlarging the scope of freedom.  The prize carries a cash award of 25 Lakh and a citation.  It was first awarded in 1986.  Additional Fact:

60 http://www.edristi.in/

 David Attenborough is an English broadcaster and natural historian.  He is best known for writing and presenting, in conjunction with the BBC Natural History Unit, the nine natural history documentary series forming the Life collection that together constitute a comprehensive survey of animal and plant life on Earth.

Link: https://www.outlookindia.com/newsscroll/british-broadcaster-david-attenborough-gets- indira-gandhi-peace-prize/1931358 Bangladesh announces national mourning in honour of former President Pranab Mukherjee Question:Which government has announced national mourning to honour the memory of the former President of India Pranab Mukherjee as “MUKTIJUDDHO SOMMANONA”: (a) Nepal (b) Bangladesh (c) Sri Lanka (d) Myanmar Answer:(b)

 Context:  The government of Bangladesh has announced national mourning to honour the memory of the former President of India Pranab Mukherjee who passed away.  Facts:  Prime Minister Sheikh Hasina said that Pranab Mukherjee was a true friend of Bangladesh who was highly revered and loved by the people of Bangladesh.  Bangladesh had conferred the honour of ‘Bangladesh Muktijuddho Sommanona’ on the former President Pranab Mukherjee in 2013 for his valuable contribution to Bangladesh’s Liberation War of 1971.  Additional Fact:  Other Indian recipients of this award were Former Prime Minister I. K. Gujral and former Prime Minister Atal Bihari Vajpayee.

Link: http://ddnews.gov.in/international/bangladesh-announces-national-mourning-honour- former-president-pranab-mukherjee

Planning & Project ADB approves USD 570 million loans for urban projects

Question : ADB has recently(September 2020) approved a loan for urban projects in Rajasthan, Madhya Pradesh.What is the loan amount ? (a) 570 million dollar (b) 670 million dollar (c) 870 million dollar (d) 1000 million dollar Answer (a) Related facts 61 http://www.edristi.in/

 The Manila-headquartered Asian Development Bank (ADB) on Friday(25 September 2020) said it has approved two loans totalling USD 570 million (about Rs 4,200 crore) for urban sector projects in Rajasthan and Madhya Pradesh. ADB said it has approved a USD 300 million loan to develop Rajasthan’s secondary towns and a USD 270 million loan for Madhya Pradesh Urban Services Improvement Project.  In a statement, ADB said the USD 300 million loan has been approved to finance inclusive water supply and sanitation infrastructure and services in secondary towns of Rajasthan.  The project is expected to build citywide access for improved water supply services for around 5,70,000 people and enhanced sanitation services for about 7,20,000 people in at least 14 secondary towns. These localities have 20,000- 1,00,000 residents  In another statement, ADB said it has approved a USD 270 million loan for construction of water supply and integrated storm water and sewage management infrastructure in Madhya Pradesh.  The loan is an additional financing intended to scale up the scope of the Madhya Pradesh Urban Services Improvement Project, which was approved in 2017 with a USD 275 million loan.

 It will expand the outcome of the current project by benefiting an additional 1,85,000 households consisting of about 1.3 million people. Link: https://economictimes.indiatimes.com/news/economy/finance/adb-approves-usd-570- million-loans-for-urban-projects-in-rajasthan-madhya- pradesh/articleshow/78322565.cms#:~:text=New%20Delhi%3A%20The%20Manila%2D headquartered,in%20Rajasthan%20and%20Madhya%20Pradesh. Haryana Orbital Rail Corridor Project Question : Cabinet has recently(September 2020) approved Haryana Orbital Rail Corridor Project from Palwal to Sonipat via Sohna-Manesar- Kharkhauda.What is the total length of the project ? (a) about 121.7 km (b) 110 km (c) 95 km (d) 80 km Anwer (a) Related facts

 The Cabinet Committee on Economic Affairs, chaired by Prime Minister Shri Narendra Modi, has given its approval to the Haryana Orbital Rail Corridor Project from Palwal to Sonipat via Sohna-Manesar-Kharkhauda. Length of the project is approximately 121.7 kilometer.  The project will be implemented by Haryana Rail Infrastructure Development Corporation Limited (HRIDC), a Joint Venture company set up by Ministry of Railways with Government of Haryana  This will facilitate diversion of traffic not meant for Delhi and will help in developing multimodal logistics hubs in Haryana State sub-region of NCR.  Estimated completion cost of the project is approximately Rs. 5,617 crore and it is proposed to be completed in 5 years  Approximately 20,000 passengers each day will be travelling through this line and 50 Million Tonnes goods traffic would also be carried out every year.

62 http://www.edristi.in/

Link: https://pib.gov.in/PressReleasePage.aspx?PRID=1654429 Indian Railways RPF disrupts illegal software “Real Mango” used to corner reservations Question:Consider the following with respect to Real Mango: (1) It is an illegal software developed for cornering confirmed Railway reservation over the IRCTC. (2) Railway Protection Force(RPF) of Indian Railways has disrupted it’s operation. Which is/are correct? (a) 1 only (b) 2 only (c) Both 1 and 2 (d) None of these Answer:(c)

 Context:  Railway Protection Force(RPF) of Indian Railways has disrupted the operation of illegal software called “Real Mango”.  Facts:  Real Mango Software:It is an illegal software developed for cornering confirmed Railway reservation over the IRCTC (Indian Railway Catering and Tourism Corporation).  How does it work? The software logs into the IRCTC server through multiple Ids.It bypasses captcha code and also gets sync with the bank OTP with the help of a mobile app and feeds it to the requisite form to book tickets automatically.  It also auto-fills the passenger and payment details in the forms and makes the whole process a quick and less time-consuming process.  This makes the complete process of Indian Rail ticket booking at a fly speed.

Link: https://pib.gov.in/PressReleasePage.aspx?PRID=1652341#:~:text=RPF%20have%20b een%20able%20to,5%20Lakhs&text=In%20a%20nation%20wide%20investigation%2C %20RPF%20has%20disrupted%20the%20operation,for%20cornering%20confirmed%2 0Railway%20reservation. Project For Conservation Of Dolphins

Question:Consider the following with respect to Gangetic river dolphin: (1) It inhabits the Ganges-Brahmaputra-Meghna and Karnaphuli-Sangu river systems of Nepal, India, and Bangladesh. (2) The animal is popularly referred to as the ‘Susu’. (3) In IUCN Red List it is critically endangered . Which is/are correct? (a) 1 and 2 (b) 2 and 3 (c) 1 and 3 (d) 1,2 and 3 Answer:(a) Context:

63 http://www.edristi.in/

 Prime Minister announced Project Dolphin on Independence Day.  Facts:  Project Dolphin:  The project is aimed at the conservation of the Gangetic Dolphins – both riverine as well as the oceanic dolphins in India.  Significance: Aquatic life is an indicator of the health of river ecosystems. As the Gangetic dolphin is at the top of the food chain, protecting the species and its habitat will ensure conservation of aquatic lives of the river.  Gangetic river dolphin:  The Gangetic river dolphin inhabits the Ganges-Brahmaputra-Meghna and Karnaphuli-Sangu river systems of Nepal, India, and Bangladesh.  Being a mammal, the Ganges River dolphin cannot breathe in the water and must surface every 30-120 seconds. Because of the sound it produces when breathing, the animal is popularly referred to as the ‘Susu’.  Population: 1,272 dolphins in Uttar Pradesh and 962 in Assam in 2019  Threats: construction of dams and barrages, and increasing pollution  IUCN Red List: Endangered  CITES: Appendix I.  Conservation Measures for Gangetic Dolphin:  Wildlife Protection Act: Gangetic Dolphin is protected under Schedule I of the Act. Further, Vikramshila Ganges Dolphin Sanctuary was established in Bihar under this Act.  Conservation Plan: The government also prepared The Conservation Action Plan for the Ganges River Dolphin 2010-2020, which “identified threats to Gangetic Dolphins and impact of river traffic, irrigation canals and depletion of prey-base on Dolphins populations”.  National Aquatic Animal: In 2009, National Ganga River Basin Authority, declared the Gangetic river dolphin as the national aquatic animal. The National Mission for Clean Ganga celebrates October 5 as National Ganga River Dolphin Day.

Link: https://indianexpress.com/article/india/project-dolphin-announced-by-pm-modi- javadekar-launch-6558304/ Pradhan Mantri Kisan SAMPADA Yojana

Question:Consider the following with respect to Pradhan Mantri Kisan SAMPADA Yojana : (1) It is a centrally sponsored scheme aimed at creating modern infrastructure with efficient supply chain management from farm gate to retail outlet. (2) The scheme will be implemented by the Ministry of Food Processing Industries(MoFPI). Which is/are correct? (a) 1 only (b) 2 only (c) Both 1 and 2 (d) None of these Answer:(b)

64 http://www.edristi.in/

 Context:  Government gave its nod to 27 projects today under the Pradhan Mantri Kisan SAMPADA Yojana for development of integrated cold chains and value addition infrastructure in the country.  Facts:  Pradhan Mantri Kisan SAMPADA Yojana: It is a Central Sector Scheme aimed at creating modern infrastructure with efficient supply chain management from farm gate to retail outlet.  Objectives:  Creation of modern infrastructure for food processing mega food parks/ clusters and individual units  To create effective backward and forward linkages – linking farmers, processors and markets  To create robust supply chain infrastructure for perishables  Implementation: The scheme will be implemented by the Ministry of Food Processing Industries(MoFPI).  Components: Seven component schemes under PMKSY:  Mega Food Parks.  Integrated Cold Chain and Value Addition Infrastructure.  Infrastructure for Agro-Processing Clusters.  Creation of Backward and Forward Linkages.  Creation/Expansion of Food Processing & Preservation Capacities.  Food Safety and Quality Assurance Infrastructure.  Human Resources and Institutions.

Link: http://newsonair.com/Main-News-Details.aspx?id=398784#:~:text=Govt.- ,gives%20nod%20to%2027%20integrated%20cold%20chain,under%20PM%20Kisan% 20SAMPADA%20Yojana&text=The%20new%20integrated%20cold%20chain,benefit%2 0nearly%202%2C57%2C000%20farmers.

Treaty & Agreements PFC – Govt. of India agreement

Question : Power Finance Corporation Ltd (PFC) has recently(29 September)signed a performance-based ‘Memorandum of Understanding’ (MoU) with Ministry of Power, Govt. of India detailing various targets to be achieved by PFC during FY 2020-21. Which of the following target is involved in this agreement ? (a) Govt. of India has set an ambitious revenue target of Rs. 36,000 Crs (b) Govt. of India has set an ambitious revenue target of Rs. 46,000 Crs (c) Govt. of India has set an ambitious revenue target of Rs. 56,000 Crs (d) Govt. of India has set an ambitious revenue target of Rs. 66,000 Crs Answer a Related facts

 Power Finance Corporation Ltd (PFC) has recently(29 September)signed a performance-based ‘Memorandum of Understanding’ (MoU) with Ministry of

65 http://www.edristi.in/

Power, Govt. of India detailing various targets to be achieved by PFC during FY 2020-21.  Govt. of India has set an ambitious revenue target of Rs. 36,000 Crs along with various performance-related parameters such as Operating Profit as percentage of Revenue from Operation, PAT as a percentage of Average Networth and Non- financial parameters viz., IPDS-related parameters.  PFC has been delivering exemplary performance from the past years and the ratings from the Government of India stands testimony to the performance.

 Incorporated on July 16th, 1986, Power Finance Corporation Ltd. is a Schedule- A Navratna CPSE, and is a leading Non-Banking Financial Corporation in the Country.

Link: https://pib.gov.in/PressReleasePage.aspx?PRID=1660015 NFL Vijaipur – ITI Raghogarh (MP ) partnership

Question : National Fertilizers Limited(NFL), a PSU under the Department of Fertilizers, has tied up with Industrial Training Institute (ITI) Raghogarh (M.P. ) to train youth in various trades and to enhance the chances of their employability in heavy and process industry.On which date NFL was incorporated ? (a) 23rd August 1974 (b) 23rd August 1976 (c) 23rd August 1978 (d) 23rd August 1980 Answer (a) Related facts

 National Fertilizers Limited, a PSU under the Department of Fertilizers, has recently(29 September 2020) tied up with Industrial Training Institute (ITI) Raghogarh (M.P. ) to train youth in various trades and to enhance the chances of their employability in heavy and process industry.  NFL, Vijaipur Unit has signed an MoU with nearby ITI, Raghogarh, to train youth in Electrician, Fitter and Welder trades.  The students will be skilled under Dual System of Training Scheme under which they will learn theoretical skills in the institute and 6 months on-the-job training in NFL Vijaipur plant.

 Overall duration of training shall be as per the course curriculum of ITI. However, the trainees will get 6 months of exposure/ training of industry (in NFL Vijaipur Unit) out of the overall duration of ITI training.  The company plans to explore more such options in future to give impetus to Skill India by training more youth from institutes around its plants.  NFL has five gas based Ammonia-Urea plants in Nangal and Bathinda plants in Punjab, Panipat plant in Haryana and two plants at Vijaipur in Guna District of Madhya Pradesh.  NFL was incorporated on 23rd August 1974.

Link: https://pib.gov.in/PressReleasePage.aspx?PRID=1660035

66 http://www.edristi.in/

Visa – IfundWomen partnership

Question : Visa has announced the launch of a grant program in India to support women entrepreneurs with its partner, IFundWomen. Which of the following statement is correct regarding this program ? (a)last date of submission of application form is October 20 (b) participants need to submit details about their business (c)three winners will be selected from the shortlist and receive grants of Rs 7,00,000 each (d)All above Answer (d) Related facts

 Visa has recently(September 2020) announced the launch of a grant program in India to support women entrepreneurs with its partner, IFundWomen.  As an extension of two programs launched in the US this year, women entrepreneurs in India will now receive funding from Visa to grow and expand their business.  IFundWomen is the marketplace for women-owned businesses and all the people who want to support them with access to capital, coaching and connections.  Visa’s partnership with IFundWomen is designed to help women entrepreneurs secure the funding they need through a series of grants.

Program

 Visa will be seeking applications from women entrepreneurs across all sectors till October 20.  To participate, applicants need to submit details about their business and online presence, along with a short video about their business.  Three winners will be selected from the shortlist and receive grants of Rs 7,00,000 each and coaching from leaders within the Visa and IFundWomen network.  Visa has partnered with FLO (the ladies’ wing of FICCI) and Instamojo as Digitization partner for the program in India.

Link: https://economictimes.indiatimes.com/small-biz/entrepreneurship/visa-extends-global- partnership-with-ifundwomen-to-entrepreneurs-in- india/articleshow/78230932.cms?from=mdr#:~:text=MUMBAI%3A%20Visa%20has%20 announced%20the,entrepreneurs%20with%20its%20partner%2C%20IFundWomen.&te xt=Visa’s%20partnership%20with%20IFundWomen%20is,through%20a%20series%20o f%20grants. India – Denmark pact

Question : On which date India and Denmark signed a Memorandum of Understanding (MoU) to increase cooperation in the area of intellectual property rights ? (a)26th September 2020 (b)28th August 2020 67 http://www.edristi.in/

(c)5th September 2020 (d)15th September 2020 Answer (a) Related facts

 India and Denmark on Saturday inked a Memorandum of Understanding (MoU) to increase cooperation in the area of intellectual property rights (IPRs) by exchanging best practices and collaborating in training programmes.  The two sides will draw up a biennial work plan to implement the MoU which will include detailed planning for carrying out cooperation activities, including scope of action, the Commerce and Industry Ministry said in a statement.  The MoU aims to increase IP cooperation between the two countries by way of exchange of best practices, experiences and knowledge on IP awareness among authorities, businesses and research and educational institutions, and collaboration in training programmes, exchange of experts, technical exchanges and outreach activities.  Both the sides would also exchange information and best practices on processes for disposal of applications for patents, trademarks, industrial designs and Geographical Indications, as also for protection, enforcement and use of IP rights.  Parties to the agreement

 The MoU was signed between DPIIT(Department for Promotion of Industry and Internal Trade) and Danish Patent and Trademark Office, Ministry of Industry, Business and Financial Affairs, Kingdom of Denmark. Link: https://economictimes.indiatimes.com/news/economy/foreign-trade/india-denmark-ink- pact-to-increase-cooperation-on-iprs/articleshow/78332502.cms Germany joins Indo-Pacific club

Question:Recently which country has joined the Indo-Pacific club? (a) Brazil (b) South Africa (c) Germany (d) None of these Answer:(c)

 Context:  Germany has joined the Indo-Pacific club. The move is aimed at securing the trade routes from aggressive China.  Facts:  Germany has drafted a 40-page guidelines as it formally adopts a strategy for the Indo-Pacific region.  The move makes it the second European nation after France to formally adopt a strategy for the Indo-Pacific region.  The strategy is designed to allow Germany to make an active contribution to shaping the international order in the Indo-Pacific.  The move by Germany augurs well for the Quad which is constituted by the United States, Japan, Australia, and India.

68 http://www.edristi.in/

Link: http://www.newsonair.com/News?title=Germany-joins-Indo-Pacific-club-aimed-at- securing-trade-routes-from-aggressive- China&id=399101#:~:text=The%20move%20is%20aimed%20at,for%20the%20Indo%2 DPacific%20region.&text=The%20strategy%20is%20designed%20to,order%20in%20th e%20Indo%2DPacific. ICICI Lombard & Yes Bank tie up Question : Private sector general insurer ICICI Lombard General Insurance Company Ltd has recently(September 2020) tied-up with Yes Bank for selling its insurance products.Which of the following statement is correct regarding Indian insurance sector ? (a) Overall insurance penetration (premiums as per cent of GDP) in India reached 3.69 per cent in 2017 from 2.71 per cent in 2001. (b) As per Union Budget 2019-20, 100 per cent foreign direct investment (FDI) was permitted for insurance intermediaries. (c) IRDAI has allowed insurers to invest up to 10 per cent in additional tier 1 (AT1) bonds that are issued by banks to augment their tier 1 capital, in order to expand the pool of eligible investors for the banks. (d) All above Answer (d) Related facts

 Private sector general insurer ICICI Lombard General Insurance Company Ltd has recently(September 2020) tied-up with Yes Bank for selling its insurance products.  This tie-up is a bancassurance.  “The partnership will provide the bank’s diverse customer base access to the insurer’s portfolio and make insurance more accessible to its customers across India,” ICICI Lombard General Insurance Company said in a regulatory filing.  Bancassurance is an agreement between a bank and an insurer to sell its offerings to the lender’s customers.  Through this tie-up, ICICI Lombard aims to offer innovative insurance solutions to Yes Bank’s customers across 28 states and 8 union territories.

Indian Insurance sector some key points

 Overall insurance penetration (premiums as per cent of GDP) in India reached 3.69 per cent in 2017 from 2.71 per cent in 2001.  As per Union Budget 2019-20, 100 per cent foreign direct investment (FDI) was permitted for insurance intermediaries.  IRDAI has allowed insurers to invest up to 10 per cent in additional tier 1 (AT1) bonds that are issued by banks to augment their tier 1 capital, in order to expand the pool of eligible investors for the banks.

Link: https://economictimes.indiatimes.com/industry/banking/finance/insure/icici-lombard- inks-bancassurance-tie-up-with-yes-bank-for-selling-insurance- products/articleshow/78100232.cms#:~:text=NEW%20DELHI%3A%20Private%20secto

69 http://www.edristi.in/ r%20general,for%20selling%20its%20insurance%20products.&text=a%20regulatory%2 0filing.- ,Bancassurance%20is%20an%20agreement%20between%20a%20bank%20and%20a n%20insurer,offerings%20to%20the%20lender’s%20customers. India to receive development assistance loan worth Rs 3,500 Cr from Japan to fight COVID-19 QuestionWhich country has committed an official Development Assistance loan of 3,500 crore rupees for the COVID-19 Crisis Emergency Response Support to India? (a) USA (b) Australia (c) Japan (d) UK Answer : (c)

 Context:  Japan has committed an official Development Assistance loan of 3,500 crore rupees for the COVID-19 Crisis Emergency Response Support to India.  Facts:  Japan International Cooperation Agency, JICA is a governmental agency that coordinates Official Development Assistance for the Government of Japan.  This programme loan aims to support India’s efforts in fighting COVID-19 and to prepare the health system to manage future epidemics.  India and Japan have had a long and fruitful history of bilateral development cooperation since 1958. The recent step further consolidates and strengthens the Strategic and Global Partnership between India and Japan.

Link: http://newsonair.com/News?title=India-to-receive-development-assistance-loan-worth- Rs-3%2C500-Cr-from-Japan-to-fight-COVID-19&id=398748 PAG – Edelweiss partnership

Quesiton : Which one of the following statement is incorrect regarding PAG & Edelweiss Partnership ? (a) PAG is one of the world’s largest Asia-focused investment groups (b) Edelweiss Group is India’s leading diversified financial services conglomerate (c) Both announced PAG’s strategic investment of approximately USD 300 million for a 31% stake in Edelweiss Wealth Management (d) Both announced PAG’s strategic investment of approximately USD 300 million for a 51% stake in Edelweiss Wealth Management Answer (c) Related facts

 PAG is one of the world’s largest Asia-focused investment groups.  Edelweiss Group is India’s leading diversified financial services conglomerate.  Both announced PAG’s strategic investment of approximately USD 300 million for a 51% stake in Edelweiss Wealth Management. 70 http://www.edristi.in/

 Indian financial services : Some key facts

 India has a diversified financial sector undergoing rapid expansion, both in terms of strong growth of existing financial services firms and new entities entering the market.

Market Size — Mutual Fund industry’s AUM grew from Rs 10.96 trillion (US$ 156.82 billion) in October 2014 to Rs 23.93 trillion (US$ 339.55 billion) in April 2020. Inflow in India’s mutual fund schemes via the Systematic Investment Plan (SIP) route reached Rs 82,453 crore (US$ 11.70 billion) in 2019. Equity mutual funds registered a net inflow of Rs 8.04 trillion (US$ 114.06 billion) by end of December 2019. India is expected to be the fourth largest private wealth market globally by 2028. Link: https://cdn1.edelweissfin.com/wp-content/uploads/2020/08/Edelweiss-PAG-Release- final-India.pdf

Conference 8th East Asia Summit Question:Consider the following with respect to East Asia Summit: (1) It was established in 2008 and is a forum of 18 countries. (2) It is a forum of strategic dialogue and cooperation on the key political, security, and economic challenges facing the Indo-Pacific region. (3) India is it’s member. Which is/are correct? (a) 1 and 2 (b) 2 and 3 (c) 1 and 3 (d) 1,2 and 3 Answer:(b)

 Context:  The 8th East Asia Summit (Economic Ministers Meeting) have stressed the importance of further strengthening regional economic cooperation and accelerating post-COVID economic growth.  Facts:  East Asia Summit: It was established in 2005.It is a forum of 18 countries.  Objective: It is a forum of strategic dialogue and cooperation on the key political, security, and economic challenges facing the Indo-Pacific region.  Members: It comprises the ten member states of the ASEAN countries along with 8 members Australia, China, Japan, India, New Zealand, the Republic of Korea, Russia and the United States.India is a founding member of the East Asia Summit.  Priority Areas: There are six priority areas of regional cooperation within the framework of the EAS which are :

71 http://www.edristi.in/

(a) Environment and Energy (b) Education (c) Finance (d) Global Health Issues and Pandemic Diseases (e) Natural Disaster Management (f) ASEAN Connectivity

 First Summit: It was held in Kuala Lumpur,Malaysia in 2005.

Link: https://asean.org/storage/2020/08/JMS-of-the-8th-EAS-EMM.pdf

Year, Day & Week World Heart Day

Question:When is the World Heart Day observed? (a) September 17 (b) September 20 (c) September 14 (d) September 29 Answer: (d) Related facts:

 Every year September 29 is observed as World Heart Day.  The day aims at drawing people’s attention to heart ailments and the whole range of associated health issues.  This initiative was launched in 2000 by the World Heart Federation as an annual event and gets celebrated on 29th day of September ever since.  The theme for World Heart Day 2020: “Use Heart To Beat Cardiovascular Disease”.  On an average, more than 17 million people die from heart-related illnesses every year.

Links: http://newsonair.com/News?title=World-Heart-Day-being-observed-today&id=400979

International Day of Peace

Question:When is the International Day of Peace observed? (a) September 17 (b) September 20 (c) September 14 (d) September 21 Answer: (d) Related facts:

 International Day of Peace is observed annually on 21 September.

72 http://www.edristi.in/

 The International Day of Peace was established in 1981 by the United Nations General Assembly.  Two decades later, in 2001, the General Assembly unanimously voted to designate the Day as a period of non-violence and cease-fire.  The UN General Assembly has declared this as a day devoted to strengthening the ideals of peace, through observing 24 hours of non-violence and cease-fire.  The 2020 theme for the International Day of Peace is “Shaping Peace Together.”

Links: https://www.un.org/en/observances/international-day-peace World Bamboo Day

Question:When is the World Bamboo Day observed? (a) September 17 (b) September 20 (c) September 14 (d) September 18 Answer: (d) Related facts:

 On 18th September 2020, World Bamboo Day was celebrated across the globe.  The day aims to promote bamboo plantation. Bamboo is a versatile group of plants which is capable of providing ecological, economic and livelihood security to the people.  The day was officially established by World Bamboo organization at the 8th World Bamboo Congress held in Bangkok in 2009.  World Bamboo Organization:

 It was set up at the 1992 International Bamboo Congress in Japan.  It is an international coordinating body for bamboo practitioners dedicated to promoting the use of bamboo and bamboo products for the sake of the environment and economy.  National Bamboo Mission (NBM):

 The restructured National Bamboo Mission was launched in 2018-19.  The Mission envisages promoting holistic growth of bamboo sector by adopting area-based,regionally differentiated strategy and to increase the area under bamboo cultivation and marketing.  Earlier NBM was launched in 2006 as a Centrally Sponsored Scheme.  India has the highest area (13.96 million hectares) under bamboo.  It is the second richest country after China in terms of bamboo diversity with 136 species (125 indigenous and 11 exotic).

Links: https://worldbamboo.net/world-bamboo-day International Day to Protect Education from Attack Question:The first-ever International Day to Protect Education from Attack is being celebrated on : (a) 8 September 73 http://www.edristi.in/

(b) 9 September (c) 10 September (d) 11 September Answer:(b)

 Context:  The first-ever International Day to Protect Education from Attack is being celebrated on September 9, 2020 under the theme “Protect Education, Save a Generation.”  Facts:  The day was established by a unanimous decision of the UN General Assembly, calling on UNESCO and UNICEF to raise awareness of the plight of millions of children living in countries affected by conflict.  Over the past five years, there have been more than 11,000 reported attacks on education in over 36 countries, according to the Global Coalition to Protect Education from Attack (GCPEA). 22,000 students, teachers and academics were killed, injured, arrested or otherwise harmed in attacks between 2015-2019.  UNESCO and UNICEF will facilitate the annual observance of the Day in close collaboration with partners within and outside the UN system.

Link: https://www.un.org/en/observances/protect-education-day International Day for the Preservation of the Ozone Layer Question: International Day for the Preservation of the Ozone Layer is known as World Ozone Day. When is the World Ozone Day celebrated every year? (a) 14 September (b) 16 September (c) 20 September (d) 18 September Answer: (b) Related facts: World Ozone Day is celebrated every year on 16 September. The theme of this year was ‘Ozone for life 35 years of ozone layer protection’. Scientific Assessment of Ozone Depletion :

 The latest Scientific Assessment of Ozone Depletion completed in 2018, shows that, as a result, parts of the ozone layer have recovered at a rate of 1-3% per decade since 2000.  At projected rates, Northern Hemisphere and mid-latitude ozone will heal completely by the 2030s. The Southern Hemisphere will follow in the 2050s and Polar Regions by 2060.  Ozone layer protection efforts have also contributed to the fight against climate change by averting an estimated 135 billion tonnes of carbon dioxide equivalent emissions, from 1990 to 2010.  There is a need to support the Kigali Amendment to the Montreal Protocol, which entered into force on 1 January 2019. By phasing down hydrofluorocarbons (HFCs), which are potent climate-warming gases, this amendment can avoid up

74 http://www.edristi.in/

to 0.4°C of global temperature rise by the end of the century, while continuing to protect the ozone layer.  Case of India:

Recent findings of a study mentions that unclean air from the Indo-Gangetic plain and central India is messing with ozone levels in the Indian subcontinent. There is a need for pan-India co-ordination to control the lung-damaging greenhouse gas, the chief ingredient of smog. There is an emergence of central India as a significant source of dirty air. Breathing air thick with ozone can have adverse impacts on human health, animals and vegetation. It also mentions that “more than a million Indians will die prematurely” each year from long-term ozone exposure by 2050. Link: https://www.un.org/en/observances/ozone-day International Literacy Day

Question:International Literacy Day is observed on: (a) 7 September (b) 8 September (c) 9 September (d) 10 September Answer:(b)

 Context:  Countries across the world are celebrating International Literacy Day on September 8, 2020.  Facts:  International Literacy Day-2020 will focus on Literacy teaching and learning in the COVID-19 crisis and beyond with a focus on the role of educators and changing pedagogies.  This day was declared International Literacy Day by UNESCO on 26th October 1966 at the 14th Session of UNESCO’s General Conference.  It was celebrated for the first time in 1967 and its main aim is to highlight the importance of literacy to individuals, communities and societies.  Additional Fact:  Despite progress made, literacy challenges persist with at least 773 million adults worldwide lacking basic literacy skills today.  According to the UNESCO’s ‘Global Monitoring Report on Education for All’ (2006), South Asia has the lowest regional adult literacy rate, at 58.6 per cent and the causes for this illiteracy range from severe poverty and the prejudice against women.

Link: https://www.un.org/en/observances/literacy-day International Day of Charity QuestionThe International Day of Charity was observed on: (a) 5 September 75 http://www.edristi.in/

(b) 6 September (c) 7 September (d) 8 September Answer:(a) Context:

 The International Day of Charity was observed on September 5, 2020.  Facts:  The International Day of Charity was established with the objective of sensitizing and mobilizing people, NGOs, and stakeholders all around the world to help others through volunteer and philanthropic activities.  The date of 5 September was chosen by the General Assembly of the United Nations in order to commemorate the anniversary of the passing away of Mother Teresa of Calcutta, who received the Nobel Peace Prize in 1979.  Mother Teresa, the renowned nun and missionary, was born Agnes Gonxha Bojaxhiu in 1910. In 1928 she went to India, where she devoted herself to helping the destitute.  In 1948 she became an Indian citizen and founded the order of Missionaries of Charity in Kolkota (Calcutta) in 1950, which became noted for its work among the poor and the dying in that city.  Mother Teresa died on September 5th 1997, at 87 years of age.

Link: https://www.un.org/en/observances/charity-day National Sports Day

Question:Consider the following statements with respect to National Sports Day: (1) National Sports Day is celebrated on 29th August every year to mark the birth anniversary of Major Dhyan Chand. (2) National Sports Awards which include the Rajiv Gandhi Khel Ratna award, the Arjuna award and the Dronacharya award. Which is/are incorrect? (a) 1 only (b) 2 only (c) Both 1 and 2 (d) None of these Answer : (b) Context:

 Prime Minister Narendra Modi paid tributes to Major Dhyan Chand and greeted all sports enthusiasts on National Sports Day.  Facts:  National Sports Day (also known as Rashtriya Khel Divas) is celebrated on 29th August every year to mark the birth anniversary of legendary hockey player Major Dhyan Chand.  Dhyan Chand was born in 1905 and was known for his magical hockey skills. He was part of the Indian hockey team which struck gold in the Olympics of 1928, 1932, and, 1936. 76 http://www.edristi.in/

 National Sports Awards which include the Rajiv Gandhi Khel Ratna award, the Arjuna award, the Dronacharya award, and the Dhyan Chand award, are conferred on athletes every year by the President of India at the Rashtrapati Bhavan on the National Sports Day. However, this year, the names would be announced by President Ram Nath Kovind virtually.

Link: https://pib.gov.in/PressReleaseIframePage.aspx?PRID=1649439

Miscellaneous Automated recognition of NPAs

Question : The Reserve Bank of India (RBI) has recently mandated the automation of bad-loan recognition by banks by — (a) June 30, 2021 (b) January 30, 2021. (c) April 30, 2021. (d) June 30, 2022. Answer (a) Related facts

 The Reserve Bank of India (RBI) on Monday(14 September 2020) mandated the automation of bad-loan recognition by banks by June 30, 2021.  The processes of provisioning calculation and income recognition will also have to be automated and banks will be required to upgrade their systems accordingly.  In its circular, the central bank said banks had earlier been advised to have appropriate information technology (IT) systems in place for identification of non- performing assets (NPA) and generation of related data/returns, both for regulatory reporting and banks’ own management information system (MIS) requirements.  “It is, however, observed that the processes for NPA identification, income recognition, provisioning and generation of related returns in many banks are not yet fully automated. Banks are still found to be resorting to manual identification of NPA and also over-riding the system-generated asset classification by manual intervention in a routine manner,” the RBI said.  All borrowal accounts, including temporary overdrafts, irrespective of size, sector or types of limits shall be covered in the automated IT- based system for asset classification, upgradation and provisioning processes.  Banks’ investments shall also be covered under the system.  Asset classification rules shall be configured in the system, in compliance with regulatory stipulations.  The calculation of provisioning requirements shall also be system-based as per pre-set rules for various categories of assets, value of security as captured in the system and any other regulatory stipulations issued from time to time on provisioning requirements.  In addition, income recognition and derecognition in case of impaired assets shall be system-driven and the amount required to be reversed from the income account should be obtained from the system without any manual intervention.

77 http://www.edristi.in/

 The system shall handle both downgrade and upgrade of accounts through a straight through process (STP) without manual intervention.  The system-based asset classification shall be an ongoing exercise for both downgradation and upgradation of accounts.

Link: https://www.financialexpress.com/industry/banking-finance/rbi-mandates-automated- recognition-of-npas-provisioning-by-june- 2021/2083130/#:~:text=The%20Reserve%20Bank%20of%20India,to%20upgrade%20th eir%20systems%20accordingly. Ban on export of onion

Question : India has recently prohibited the export of all varieties of onion except those cut, sliced or in powder form. Which body is responsible for this restrictions ? (a) The Directorate General of Financial Trade (b) Committee on financial stability (c) Export-Import Bank (d) None of the above Answer (d) Related Fact :

 India has recently(14 September 2020) prohibited the export of all varieties of onion except those cut, sliced or in powder form.  According to a notification of the Directorate General of Foreign Trade (DGFT) export ban covers Bangalore Rose and Krishnapuram varieties of onions which are largely exported.  Background  India had last year banned the exports of the key kitchen staple in the wake of its rising prices in the country and set the minimum export price (MEP) of onion at $850 a tonne to curb its shipments and help bring down soaring prices in the domestic market due to floods in parts of major growing states of Maharashtra and Karnataka.  The MEP was lifted in March this year and onions were made free for export.  A huge amount of exports were done in the last few months during the Covid-19 pandemic  India exported $198 million of onions in the April-June period of FY21 and $440 million in the entire 2019-20.  Bangladesh, Malaysia, UAE and Sri Lanka are the top importers of Indian onions.  Dgft  The Directorate General of Foreign Trade (DGFT) is the agency of the Ministry of Commerce and Industry of the Government of India responsible for administering laws regarding foreign trade and foreign investment in India.

Link: https://economictimes.indiatimes.com/news/economy/agriculture/india-bans-export-of- all-varieties-of-onion/articleshow/78109131.cms IDC India Research : Degrowth in India’s external storage market

78 http://www.edristi.in/

Question: Which of the following statement is correct regarding India’s external storage market? (a) India’s external storage market witnessed degrowth of 36.8 per cent year-over-year by vendor revenue (b) Market stood at USD 63.8 million in the April-June 2020 quarter (c)The majority of the y-o-y decline is due to the impact of COVID-19 resulting in delayed demand for storage across organisations (d) all above Answer- d Related facts

 India’s external storage market witnessed degrowth of 36.8 per cent year-over- year by vendor revenue and stood at USD 63.8 million in the April-June 2020 quarter, research firm IDC said on Tuesday(29September 2020).  The majority of the y-o-y decline is due to the impact of COVID-19 resulting in delayed demand for storage across organisations during the quarter  The external enterprise storage systems market is expected to grow at a single- digit compounded annual growth rate (CAGR) for the 2019-2024 time period  Dell Technologies led the market with 20.1 per cent market share by vendor revenue, followed by Hewlett Packard Enterprise (HPE) with a 19 per cent market share in June 2020 quarter.  External storage  External storage, also called as auxiliary storage or secondary storage, comprises of all addressable data storage that do not belong to the computer’s main storage or memory.  Such devices are not permanently fixed inside a computer.  They offer advantages such as enhanced data storage capacity, portability, and data backup options.

Link: https://m.economictimes.com/tech/software/indias-external-storage-market-sees-37-pc- decline-in-growth-on-covid-19-woes- idc/amp_articleshow/78389251.cms#aoh=16014035893982&csi=1&referrer=https%3A %2F%2Fwww.google.com&_tf=From%20%251%24s Demerger of the Textiles Business ofGHCL into its wholly owned subsidiary “GHCLTextiles” Question: Under which act CCI has approved demerger of the textiles business of GHCL Ltd. (“GHCL”)into its wholly owned subsidiary“GHCL Textiles” ? (a) Section 31(1) of the Competition Act, 2002. (b) Section 21(1) of the Competition Act, 2002. (c)Section 11(1) of the Competition Act, 2002. (d) Section 41(1) of the Competition Act, 2002. Answer- (a) Related facts Context

79 http://www.edristi.in/

 CCI has recently (September 2020) approved demerger of the textiles business of GHCL Ltd.(“GHCL”) into its wholly owned subsidiary “GHCLTextiles” under Section 31(1) of the Competition Act, 2002.  GHCL

 GHCL is a public limited company incorporated in India and is stated to be engaged in the business of (i) manufacture and sale of inorganic chemicals including but not limited to Soda Ash (Dense grade and Light grade), Sodium Bicarbonate, Industrial Salt and Consumer Products (‘Chemical Business’) and (ii) manufacture and sale of textiles including but not limited to yarn manufacturing along with weaving, processing, cutting and sewing of home textiles products (‘Textiles Business’).  GHCL Textiles

 GHCL Textiles is a Public Limited Company incorporated in India. Presently, GHCL Textiles Ltd. is a wholly owned subsidiary of GHCL.  It is a newly incorporated company and has not initiated any business activity.  Pursuant to the proposed demerger, GHCL will retain its chemicals and consumer products business and GHCL Textiles will have the demerged Textiles business.  CCI

 The Competition Act, 2002, as amended by the Competition (Amendment) Act, 2007, follows the philosophy of modern competition laws.  The objectives of the Act are sought to be achieved through the Competition Commission of India(CCI), which has been established by the Central Government with effect from 14th October 2003.

Link: https://pib.gov.in/PressReleasePage.aspx?PRID=1660138 NSE : New version of RFQ platform for debt securities Question : To cater to requirements of bond markets, leading stock exchange NSE has recently launched the new version of Request for Quote (RFQ) platform.Which of the following statement is correct regarding RFQ ? (a) The RFQ platform for debt securities was launched in June. (b) It allows market participants to transact in cash securities (c) New version of RFQ platform has several features which include voice-based negotiations in order to bring transparency to bond trading (d) None of the above Answer (c) Related facts

 To cater to requirements of bond markets, leading stock exchange NSE has recently(28 September 2020) launched the new version of Request for Quote (RFQ) platform.  The RFQ platform for debt securities was launched in February that allows market participants to transact in debt securities by requesting quotes from several participants, simultaneously.

80 http://www.edristi.in/

 Now, the bourse has launched the new version with several features which include voice-based negotiations in order to bring transparency to bond trading, as per the statement by the National Stock Exchange (NSE).  To reduce information asymmetry, the NSE RFQ platform provides information about bonds, term sheets, price information, market quotes, among others to all market players to trade in corporate bonds, securitized debt, municipal bonds, commercial paper, government securities (G-Sec) and treasury bills (T-bills).  RBI regulates money markets & G-secs; while SEBI regulates the Corporate debt market & bond markets.

The Indian debt market, primarily of the fixed-income variety, can be broadly classified into: Money Market,Bank and Corporate Deposits,Government Securities,Corporate & PSU Bond Market. Link: https://economictimes.indiatimes.com/markets/stocks/news/nse-launches-new-version- of-rfq-platform-for-debt-securities/articleshow/78368458.cms

Kalpataru Power Transmission Ltd, JV partner complete 74 pc equity stake sale in JKTPL

Question : KPTL(Kalpataru Power Transmission Ltd) had recently(May 2020) announced that it would sell the joint venture at an enterprise value of rupees – (A)310 crore (b)450 crore (c)650 crore (d)920 crore Answer (a) Related facts

 Kalpataru Power Transmission Ltd (KPTL) on Monday(28 September 2020) said the firm and its joint venture partner have completed 74 per cent equity stake sale in Jhajjar KT Transco Pvt Ltd (JKTPL) to IndiGrid.  In May this year, KPTL had announced execution of share purchase agreement (SPA) to sell its entire stake in JKTPL to India Grid Trust (IndiGrid) for a total Enterprise Value (EV) of JKTPL at about Rs 310 crore The KPTL held 49.72 per cent, Klassik Vinyl Products LLP (KVPL) held 1.32 per cent and Techno Electric & Engineering Company Ltd (Techno) held balance 48.96 per cent equity in JKTPL.

The sale of balance shares held by KPTL and Techno is expected to be completed shortly. Link: https://economictimes.indiatimes.com/industry/energy/power/kalpataru-power- transmission-ltd-jv-partner-complete-74-pc-equity-stake-sale-in- jktpl/articleshow/78369117.cms

81 http://www.edristi.in/

NSE, SGX ink formal pact on NSE IFSC-SGX Connect

Question : National Stock Exchange (NSE) and Singapore Exchange (SGX) have recently entered into a formal agreement to cement the key terms for operationalising the NSE IFSC-SGX Connect. Which one of the following statement is correct regarding NSE IFSC–SGX Connect ? (a) Proposed NSE IFSC–SGX Connect got regulatory nod in August 2019 (b) Proposed NSE IFSC–SGX Connect got regulatory nod in August 2018 (c) Proposed NSE IFSC–SGX Connect got regulatory nod in December 2019 (d) Proposed NSE IFSC–SGX Connect got regulatory nod in December 2019 Answer (a) Related facts

 National Stock Exchange (NSE) and Singapore Exchange (SGX) said on Tuesday(22 September 2020) they have entered into a formal agreement to cement the key terms for operationalising the NSE IFSC-SGX Connect.  The two exchanges will also withdraw arbitration proceedings.  The parties did not elaborate on the terms and structure of the formal agreement, but said the proposed NSE IFSC- SGX Connect will bring together international and Gujarat International Finance Tec-City (GIFT) participants to create a bigger liquidity pool for Nifty products in GIFT City.  In August 2019, the two exchanges had received a set of approved regulatory dispensations from their statutory regulators obtaining their support on a joint proposal that the two exchanges submitted earlier this year.  On Tuesday, the two exchanges said they have recently received further regulatory clarifications from the relevant authorities on implementation of the Connect, and they will continue to work with key stakeholders to develop infrastructure and ensure member readiness prior to its implementation.

Link: https://economictimes.indiatimes.com/markets/stocks/news/nse-sgx-ink-formal-pact-on- nse-ifsc-sgx-connect/articleshow/78254079.cms Bandhan bank : new vertical “Emerging Entrepreneurs Business”

Question : Which bank has launched “Emerging Entrepreneurs Business Vertical”? A. Central Bank B. Bandhan Bank C. State Bank D. Allahabad Bank Answer (a) Related facts

 Bandhan Bank has recently launched a new vertical “Emerging Entrepreneurs Business”.  The bank on Wednesday(16 September,2020) announced the appointment of Kumar Ashish as the head of this new division.  In line with the Bank’s business strategy, a new vertical called Emerging Entrepreneurs Business (EEB) has been formed to serve the emerging and diversified needs and aspirations of this segment.  “emerging entrepreneurs business” includes its microbanking division, micro home loan and micro enterprise loans.

82 http://www.edristi.in/

Bandhan Bank : Background

 After completing five years as a universal bank and being successful in building a well-diversified loan book, Bandhan Bank’s key focus still lies with partnering the unbanked and the underbanked in their journey of becoming entrepreneurs.

Link: https://economictimes.indiatimes.com/industry/banking/finance/banking/bandhan-bank- appoints-kumar-ashish-to-head-emerging-entrepreneurs-business- vertical/articleshow/78148039.cms

Requirement Climate Risk Report Question: In which of the following country, it will become mandatory for financial sector to report on climate risks, if the law regarding this approved by its parliament? a) Australia b) New Zealand c) Japan d) India Answer: (b) Related Facts:-

 On 15 September 2020 New Zealand Government announced that New Zealand will be the first country in the world to require the financial sector to report on climate risks,  The announcement regarding such provisions came from James Shaw Minister for Climate Change, New Zealand.  He informed that the changes build on the huge progress the New Zealand Government has made to tackle the climate crisis.  The minister said that this is another step on the journey the Government of New Zealand is taking towards a low carbon future for Aotearoa and a cleaner, safer planet for future generations.  It should be noted that Aotearoa is another name of New Zealand.  Many large businesses in New Zealand do not currently have a good understanding of how climate change will impact on what they do.  The minister said that, the changes will bring climate risks and resilience into the heart of financial and business decision making.  James Shaw said that it would ensure the disclosure of climate risk is clear, comprehensive and mainstream,  The new regime will be on a comply-or-explain basis, based on the Task Force on Climate-related Financial Disclosures (TCFD) framework, which is widely acknowledged as international best practice.  If approved by Parliament, financial entities could be required to make disclosures in 2023 at the earliest.  Businesses covered by the requirements will have to make annual disclosures.  These disclosers will cover governance arrangements, risk management and strategies for mitigating any climate change impacts.  If businesses are unable to disclose, they must explain why.  In total, around 200 organisations will be required to disclose their exposure to climate risk.  This includes large Crown Financial Institutions, such as ACC and the NZ Super Fund. 83 http://www.edristi.in/

 The basic concept behind this new rule is that, if businesses know how climate change will impact them in the future they can change and adopt low carbon strategies.  COVID-19 has highlighted how important it is that we plan for and manage systemic economic shocks – and there is no greater risk than climate change.  The new climate reporting requirements will apply to:  All registered banks, credit unions, and building societies with total assets of more than $1 billion  All managers of registered investment schemes with greater than $1 billion in total assets under management  All licensed insurers with greater than $1 billion in total assets under management or annual premium income greater than $250 million  All equity and debt issuers listed on the NZX  Crown financial institutions with greater than $1 billion in total assets under management, such as ACC and the NZ Super Fund  Overseas incorporated organisations would also be required to disclose in their New Zealand annual reporting.  The $1 billion threshold will make sure about 90 per cent of assets under management in New Zealand are included within the disclosure system.  The External Reporting Board (XRB) will develop one or more reporting standards, which entities may either comply with, or if they do not comply, explain why not.  The Financial Markets Authority will be responsible for independent monitoring, reporting and enforcement.

Link:- https://www.scoop.co.nz/stories/PA2009/S00127/new-zealand-first-in-the-world-to- require-climate-risk-reporting.htm Mystery of Elephant tragedy of Botswana unfolded Question: As the test results about the death of elephants in Botswana came out. What is the reason behind the death of large number of elephant in that country? a) Shortage of Food b) Draught c) Virus d) Cyanobacteria Answer: (d) Related Facts:-

 On 21 September, 2020 Wild Life Department officials in Botswana announced the test result about mysterious death of large number of elephants in that country during July, 2020.  The test result established that toxins in water produced by cyanobacteria killed more than 300 elephants in Okavango delta in Botswana this year.  The animals were found with tusks intact, ruling out poaching and deliberate poisoning.  Wild life authorities believe the elephants could have ingested the bacteria while searching for food. The carcasses were found near water sources.

84 http://www.edristi.in/

 According to The department’s principal veterinary officer Mmadi Reuben latest tests have detected cyanobacterial neurotoxins to be the cause of deaths. Cyanobacteria:  Cyanobacteria are microscopic organisms common in water and sometimes found in soil.  Although not all of such bacteria produce toxins but scientists say toxic ones are occurring more frequently as climate change and rise in average global temperature.  Some cyanobacterial blooms can harm people and animals and scientists are concerned about their potential impact.  Climate change leads to warmer water temperatures, which many cyanobacteria prefer.  This noticeable, those other animals in the Okavango Panhandle region appeared unharmed by the killing effect of the toxins produced by cyanobacteria.

Link: https://www.thehindu.com/news/international/bacteria-behind-deaths-of-330-botswana- elephants/article32663416.ece Possible sign of Microbial life on Venus detected Question: Recent discovery of which of the following gas in the clouds of the planet Venus enhanced the possibility of microbial life in its atmosphere? a) Nitrogen b) Ammonia c) Phosphine d) Hydrogen sulphide Answer: (c) Related Facts: • On14th September, 2020 Scientists announced the discovery of a rare molecule — phosphine — in the clouds of Venus. • Discovery of phosphine in the atmosphere of Venus is being considered as possible sign of microbial life by the astronomers. • A team of researchers first used the James Clerk Maxwell Telescope (JCMT) in Hawaii to detect the phosphine. • Scientists then used 45 telescopes of the Atacama Large Millimeter/sub millimeter Array (ALMA) in Chile to confirm their discovery. • In above two attempts Venus was observed at a wavelength of about 1 millimeter, much longer than the human eye can see. • It should be known that only telescopes at high altitude can detect this wavelength effectively. • According to the research published in the journal in Nature Astronomy. The detection of phosphine molecules, which consist of hydrogen and phosphorus, could point to this extra-terrestrial ‘aerial’ life.

85 http://www.edristi.in/

• The second planet from the sun, Venus has an atmosphere stifled by carbon dioxide gas. • The surface temperatures at the Venus averages more than 800°F (more than 425°C). • At Venus clouds at just 31 miles below the top of atmosphere may have the temperature 30°C and has a pressure similar to that at ground level on Earth. • Although NASA’s spacecraft Cassini detected phoshine in the atmospheres of Jupiter and Saturn, but there, they say, it formed due to immense heat and pressures which jammed the phosphorus and hydrogen atoms together to form the phosphine molecule. • On smaller, rocky planets like Earth and Venus, the researchers say, there is not enough energy to produce copious amounts of phosphine in the same way but biological process can produce it here. • This is an important reason that the researchers speculating microbial life may be the cause of phosphine presence in the Venus clouds. Link: https://www.nationalgeographic.com/science/2020/09/possible-sign-of-life-found-on- venus-phosphine-gas/ Dekho Apna Desh Webinar Series

Question:Which of the following destinations one can encounter in Punjab ‘the land of Five Waters’: (1) Baradari Garden (2) Sheesh Mahal (3) Virasat-e-Khalsa (4) Wagah border Which is/are correct? (a) 1,2 and 3 (b) 2,3 and 4 (c) 1,2 and 4 (d) 1,2,3 and 4 Answer:(d)

 Context:  Ministry of Tourism organises a webinar titled “Punjab- A historic perspective” under Dekho Apna Desh Webinar Series.  Facts:  Punjab: It is called ‘the land of Five Waters’ referring to the rivers Jhelum, Chenab, Ravi, Sutlej, and Beas.  Who controlled it? Much of the frontier was occupied by Maharaja Ranjit Singh also known as Sher-e-Punjab in the early 19th century and then taken over by the East India Company when it annexed the Punjab in 1849.  Important Festivals of Punjab: Teej, Lohri, Basant Panchami, Baisakhi and Hola Mohalla.  Popular dance forms: Bhangra and Giddha.

86 http://www.edristi.in/

 Important tourist attractions: Kali Temple, Baradari Garden, Sheesh Mahal, Gurudwara Dukh Niwaran Sahib, Qila Mubarak complex, Gobindgarh fort and Wagah border.  Golden Temple: It is a major pilgrimage destination for devotees from around the world.The Gurdwara is built around a man-made pool (sarovar) that was completed by the fourth Sikh Guru, Guru Ram Das in 1577.Guru Arjan, the fifth Guru of Sikhism requested Sai Mir Mian Mohammed, a Muslim Pir of Lahore to lay its foundation stone in 1589.  Virasat-e-Khalsa: It is a museum of Sikhism which celebrates 500 years of the Sikh history and the 300th anniversary of the birth of Khalsa, based on the scriptures written by the tenth Sikh guru, Guru Gobind Singh Ji.  Partition Museum: It is a public museum located in the town hall in Amritsar, India.The museum aims to become the central repository of stories, materials and documents related to the post-partition riots that followed the division of British India into two independent countries: India and Pakistan.

Link: https://pib.nic.in/PressReleseDetailm.aspx?PRID=1652418#:~:text=Ministry%20of%20T ourism%20organises%20a,Dekho%20Apna%20Desh%20Webinar%20Series&text=Jas winder%20Singh%20started%20the%20presentation,Ravi%2C%20Sutlej%2C%20and %20Beas. Rogan art’s connect during COVID-19

Question:Consider the following with respect to Rogan Art: (1) It is an art of cloth printing practiced in the Kutch District of Gujarat. (2) The art was started by the Khatris, a Muslim community who came to India from Sindh, Pakistan. Which is/are correct? (a) 1 only (b) 2 only (c) Both 1 and 2 (d) None of these Answer:(c)

 Context:  The centuries old tradition of Rogan art is facing an unprecedented challenge due to pandemic.  Facts:  The word Rogan means “Oil” in Persian.It is an art of cloth printing practiced in the Kutch District of Gujarat.  The art was started by the Khatris, a Muslim community who came to India from Sindh, Pakistan.  In this art, Castor oil is heated and cast into cold water and the thick residue is then mixed with natural colours.Then, using a stylus or blocks, this resultant paint is meticulously transferred on to a cloth to make floral, animal (peacock) and geometric patterns.

Link:

87 http://www.edristi.in/ https://www.thehindu.com/entertainment/art/artist-jabbar-khatri-talks-about-rogan-arts- connect-during-covid-19/article32541097.ece August rainfall highest since 1926

Question:Consider th following regarding Monsoon in India: (1) August rainfall in 2020 has been the highest since 1926. (2) This year’s monsoon as a whole were likely to be within the normal (96 to 104% of the long period average) rainfall. Which is/are correct? (a) 1 only (b) 2 only (c) Both 1 and 2 (d) None of these Answer:(c)

 Context:  August rainfall this year has been the highest since 1926 with 32.7 cm — or about 27% more than what is normal for the month, according to data from the India Meteorological Department (IMD). In August 1926, the rainfall recorded was 34.8 cm, exceeding the normal by 33%.  Facts:  The heavy rain in August was due to several long-lasting low-pressure systems, or rain-bearing winds, that formed in the Bay of Bengal and were vigorous enough to travel all the way from the south-eastern coast up to north-west India.  The surplus rain was primarily in Rajasthan, Gujarat, Maharashtra, Karnataka, Kerala, Telangana, Chhattisgarh and Odisha.  IMD officials said that though August rainfall was in excess, the figure for this year’s monsoon as a whole were likely to be within the department’s June forecast of a normal (96 to 104% of the long period average) rainfall.  From June to September 6, India had registered 7% more rainfall than what is normal for this period. In the normal course, the monsoon begins its retreat from September 15 and this can go on for nearly a month.

Link: https://www.thehindu.com/news/national/august-rainfall-highest-since-1926- imd/article32546214.ece#:~:text=August%20rainfall%20this%20year%20has,exceeding %20the%20normal%20by%2033%25. Centre launches 24/7 toll-free mental health helpline Question:Consider the following with respect to KIRAN 24/7 toll-free helpline: (1) It was launched by Ministry of Home Affairs. (2) The helpline aims to provide support to people facing human and drug trafficking. Which is/are incorrect? (a) 1 only (b) 2 only (c) Both 1 and 2 (d) None of these Answer:(c) 88 http://www.edristi.in/

 Context:  Ministry of Social Justice and Empowerment launched a 24/7 toll-free mental health rehabilitation helpline called KIRAN  Facts:  The helpline aims to provide support to people facing anxiety, stress, depression, suicidal thoughts and other mental health concerns.  It can be called from landline and mobile phones across the country at the number 1800-599-0019.

Link: https://www.thehindu.com/sci-tech/health/centre-launches-247-toll-free-mental-health- helpline/article32545325.ece Air Suvidha Portal

Question:Consider the following with respect to Air Suvidha Portal: (1) It aims to provide a contactless service to international fliers who have a connecting flight from Mumbai to other states. (2) Ministry of Civil Aviation has developed this portal. Which is/are correct? (a) 1 only (b) 2 only (c) Both 1 and 2 (d) None of these Answer:(d)

 Context:  Delhi International Airport Limited(DIAL) in collaboration with the Ministry of Civil Aviation, Health and Family Welfare and External Affairs has developed a web portal named ‘AIR SUVIDHA’.  Facts:  Air Suvidha Portal: It aims to provide a contactless service to international fliers who have a connecting flight from Delhi to other states.  Features: The portal comprises two forms; a self-declaration form where passengers fill in basic details and information about recent travel history and an exemption form, if approved, will exempt passengers from institutional quarantine.  All international passengers need to fill in the self-declaration forms on the portal.Previously, it would be filled at the airport prior to departure.

Link: https://government.economictimes.indiatimes.com/news/digital-india/egovwatch-delhi- airport-launches-air-suvidha-portal-for-india-bound-travelers/77477756 Moplah Rioters Question:Consider the following with respect to Moplah rebellion: (1) It is also known as the Malabar rebellion and was an armed revolt by the Mappila Muslims of Kerala in 1921. (2) It was started as a resistance against the British colonial rule, the prevailing feudal 89 http://www.edristi.in/ system and in favour of the Khilafat Movement but ended in communal violence between Hindus and Muslims. Which is/are correct? (a) 1 only (b) 2 only (c) Both 1 and 2 (d) None of these Answer:(c)

 Context:  A report submitted to the Indian Council of Historical Research (ICHR) in 2016 had recommended the removal of the Wagon Tragedy victims and Malabar Rebellion leaders Ali Musliyar and Variamkunnath Ahmad Haji, and Haji’s two brothers from a book on martyrs of India’s freedom struggle.  Facts:  Malabar rebellion also known as the Moplah rebellion was an armed revolt by the Mappila Muslims of Kerala in 1921.  It was started as a resistance against the British colonial rule, the prevailing feudal system and in favour of the Khilafat Movement but ended in communal violence between Hindus and Muslims.  Main Leaders of the Rebellion: Variyankunnath Kunjahammed Haji, Sithi Koya Thangal and Ali Musliyar  The British government responded with aggression bringing in Gurkha regiments to suppress the rebellion and imposed martial law.  Wagon Tragedy: A noteworthy event of the British suppression was the wagon tragedy when approximately 60 Mappila prisoners on their way to prison, were suffocated to death in a closed railway goods wagon.

Link: https://www.thehindu.com/news/cities/Kochi/moplah-rioters-not-freedom-fighters- report/article32537886.ece?homepage=true GST : Twelve states have opted for the first option offered by the centre Question : Which states have opted for the first option offered by the centre — Rs 97,000 crore borrowing for compensation of revenue shortfalls due to goods and service tax (GST) transition ? (a) Andhra Pradesh, Uttar Pradesh,Uttarakhand,Bihar & Gujarat (b) Haryana, Karnataka & Madhya Pradesh (c) Odisha,Meghalaya, Sikkim & Tripura (d) All above Answer (d) Related facts

 Twelve states including Andhra Pradesh, Bihar, Gujarat, Haryana, Karnataka, Madhya Pradesh, Meghalaya, Sikkim, Tripura, Uttar Pradesh, Uttarakhand and Odisha so far have opted for the first option offered by the centre — Rs 97,000 crore borrowing for compensation of revenue shortfalls due to goods and service tax (GST) transition.

90 http://www.edristi.in/

 Manipur is the only state that has opted for the second option that entails borrowing under the Rs 2.35 lakh crore window, which entails revenue shortfall due to GST transition as also Covid19 induced economic slowdown.  Goa, Assam, Arunachal Pradesh, Nagaland, Mizoram and Himachal Pradesh are likely to give their preference among both the borrowing options provided by Centre last month, in a couple of days.  The GST revenues including the compensation cess collections have been hit hard following the pandemic.  The centre, which has additional borrowing planned for FY21, wants states to go for borrowing to meet the shortfall in the cess fund.  Under the first window, both the principal and interest would be serviced via the cess fund and states would also receive balance compensation( Covid19 revenue loss). In the second option, principal would be serviced by the cess fund.  In the second option, principal would be serviced by the cess fund.  The GST Council will now meet on October 5, instead of September 19. The issue of GST compensation shortfall may be raised in the monsoon session of Parliament that begins from today(13 September 2020).  About 10 Opposition led states have rejected both borrowing options on the grounds that the Centre should borrow instead of states, while some have also sought the Prime Minister’s intervention to resolve the burning issue.

Link: https://economictimes.indiatimes.com/news/economy/policy/13-states-submit- borrowing-option-to-centre-to-fund-gst-shortfall/articleshow/78093905.cms Ministry of Corporate Affairs : Acquisition of 20% of the issued and paid-up equity share capital of Piramal Pharma Limited

Question : The Competition Commission of India has recently(September 2020) approved proposed combination involving acquisition of 20% of the issued and paid-up equity share capital of Piramal Pharma Limited by CA Clover Intermediate II Investments (Curie). CA Clover Intermediate II Investments is an affiliated entity of — (a) CAP V Mauritius Limited (b) CAP V Spain Limited (c) CAP V Germany Limited (d) CAP V Dubai Limited Answer (a) Related facts

 The Competition Commission of India approves proposed combination involving acquisition of 20% of the issued and paid-up equity share capital of Piramal Pharma Limited by CA Clover Intermediate II Investments (Curie).  CA Clover Intermediate II Investments is an affiliated entity of CAP V Mauritius Limited.

 It is an investment fund managed and advised by affiliated entities of The Carlyle Group Inc.  Curie is a special purpose acquisition vehicle, owned and controlled by investment funds advised by the affiliates of the Carlyle Group Inc. (“Carlyle Group”).

91 http://www.edristi.in/

Link: https://pib.gov.in/PressReleaseIframePage.aspx?PRID=1653537 Microscopic images of coronavirus infected cells taken from respiratory tract Question : Scientists have produced images of the novel coronavirus infecting lab-grown respiratory tract cells, findings that illustrate the number of virus particles that are produced and released per cell inside the lungs. Which institute’s researchers captured these images ? (a) University of North Carolina (UNC) Children’s Research Institute (b) Centenary Hospital for Women and Children, Canberra (c) Queensland Children’s Hospital, Brisbane (d) Children’s National Medical Center, Washington, D.C. Answer (a) Related facts

 Scientists have recently(September 2020) produced images of the novel coronavirus infecting lab-grown respiratory tract cells, findings that illustrate the number of virus particles that are produced and released per cell inside the lungs.  The researchers, including Camille Ehre from the University of North Carolina (UNC) Children’s Research Institute, captured these images to illustrate how intense SARS-CoV-2 infection of the airways can be in very graphic and easily understood images.  The generated high-powered microscopic images show a large number of the virus particles on human respiratory surfaces, ready to spread infection across tissues, and to other people.  In the research, the scientists inoculated the novel coronavirus SARS-CoV-2 into human bronchial epithelial cells of the lungs, which they then examined 96 hours later using the high-powered scanning electron microscopy.  The images, published in the New England Journal of Medicine, were re-colorised, and show infected hairy ciliated cells with strands of mucus attached to cilia tips.

 The scientists explained that the cilia are hair-like structures on the surface of airway epithelial cells that transport mucus and trapped viruses from the lungs.  Using a higher power magnification, they showed the structure and density of SARS- CoV-2 produced by human airway epithelia.

 These virus particles, the researchers said, are the complete, infectious form of the virus released onto respiratory surfaces by infected host cells.  They said the imaging research helps illustrate the incredibly high number of virions produced and released per cell inside the human respiratory system.

 According to the scientists, the large viral burden is a source for spread of infection to multiple organs of an infected individual, and likely mediates the high frequency of COVID-19 transmission to others.

They said the images make a strong case for the use of masks by infected and uninfected individuals to limit SARS-CoV-2 transmission.

92 http://www.edristi.in/

Link: https://economictimes.indiatimes.com/news/science/scientists-publish-microscopic- images-of-coronavirus-infected-cells-taken-from-respiratory- tract/articleshow/78089057.cms Low ozone found over Brahmaputra river valley

Question:With respect to ground-level ozone, consider the following : (1) Tropospheric, or ground-level ozone, is created by chemical reactions between oxides of nitrogen (NOx) and volatile organic compounds (VOC). (2) Recently scientists have evaluated the near surface ozone in the Brahmaputra River Valley (BRV) and found relatively high concentration of Ozone over Guwahati compared to the other urban locations in India. Which is/are correct? (a) 1 only (b) 2 only (c) Both 1 and 2 (d) None of these Answer:(a) Context:

 Researchers have found that the concentration of near surface ozone in Brahmaputra River Valley (BRV) region in the North-East corner of India is low compared to the other urban locations in India.  Facts:  Tropospheric, or ground-level ozone, is created by chemical reactions between oxides of nitrogen (NOx) and volatile organic compounds (VOC).  It usually increases when pollutants emitted by cars, power plants, industrial boilers, refineries, chemical plants, and other sources chemically react in the presence of sunlight, impacting human health.  Recent study:  Scientists at the Aryabhatta Research Institute of Observational Sciences (ARIES), Nainital have evaluated the near surface ozone in the Brahmaputra River Valley (BRV) and found relatively low concentration of Ozone over Guwahati compared to the other urban locations in India.  The examination of nitric oxide, nitrogen dioxide, and ozone concentrations in this study suggested that this site is well influenced by local sources such as adjacent major national highway.  During the daylight hours, the site is in or nearly in a photo-stationary state, indicating a low impact of organic species on the ozone concentrations.

Link: https://www.hindustantimes.com/environment/air-pollution-low-ozone-layer-found-over- brahmaputra-river-valley/story-GRC0DyHjIUXvZPoTZwggwJ.html Act of God Question:Consider the following with respect to Force majeure clause: (1) It is an exception to the Law of Contract that releases the party of its obligations to

93 http://www.edristi.in/ an extent when events beyond their control take place. (2) It can be used in case of war, riots, natural disasters or acts of God, strikes, introduction of new government policy imposing an embargo, boycotts, outbreak of epidemics. Which is/are correct? (a) 1 only (b) 2 only (c) Both 1 and 2 (d) None of these Answer:(c) Context:

 Businesses are looking towards a legal provision — the force majeure or “Act of God” clause— to cut losses.  Facts:  Force majeure clause  It is an exception to the Law of Contract that releases the party of its obligations to an extent when events beyond their control take place and leave them unable to perform their part of the contract.  Conditions in which Force majeure clause can be used:  War, riots, natural disasters or acts of God, strikes, introduction of new government policy imposing an embargo, boycotts, outbreak of epidemics.  In case a contract does not have a force majeure clause, there are some protections in common law that can be invoked by parties.  For example, the Indian Contract Act, 1872 provides that a contract becomes void if it becomes impossible due to an event after the contract was signed that the party could not prevent.  Implications of force majeure clause  When the clause is triggered, parties can decide to break from their obligations temporarily or permanently without necessarily breaching the contract.  If a party to a contract believes that the other party has invoked the force majeure clause in an unjustified situation, it can move court seeking performance of the contract.  Court rulings have established that force majeure cannot be invoked when performance of the contract has become difficult, but only when it has become impossible.  Example: In April 2020, the Bombay High Court did not accept the force majeure argument in a case where the petitioner argued that Covid-19-related lockdowns had frustrated a contract for supply of steel.

Link: https://indianexpress.com/article/explained/act-of-god-covid-19-lockdown-economy- impact-6583530/ Young Child Outcomes Index Question:With respect to Young Child Outcomes Index consider the following : (1) The index measures health, nutrition and cognitive growth with the help of indicators such as infant mortality rate, stunting and net attendance at the primary school level. 94 http://www.edristi.in/

(2) Tamil Nadu and Mizoram are among the top two States for well-being of children. Which is/are correct? (a) 1 only (b) 2 only (c) Both 1 and 2 (d) None of these Answer:(a) Context:

 Mobile Creches, a non-governmental organization has released reports Young Child Outcomes Index(YCOI) and Young Child Environment Index(YCEI) as a part of its ‘State of the Young Child in India’ report.  Facts:  Young Child Outcomes Index(YCOI): The index has been constructed for two time periods (2005–2006 and 2015–2016) to enable inter-State comparisons as well as provide an idea of change over time.  Parameters: The index measures health, nutrition and cognitive growth with the help of indicators such as infant mortality rate, stunting and net attendance at the primary school level.  Key Takeaways:  Top States: Kerala, Goa, Tripura, Tamil Nadu and Mizoram are among the top five States for well-being of children.  Below Average States: It identifies eight States that have scores below the country’s average: they are Assam, Meghalaya, Rajasthan, Chhattisgarh, Madhya Pradesh, Jharkhand, Uttar Pradesh and Bihar.  Young Child Environment Index(YCEI): The index aims to understand the policy and environment enablers that influence a child’s well-being.It was constructed for 2015– 2016 only due to limitations of data availability.  Parameters: The index uses five policy enablers that influence child well-being outcomes, including poverty alleviation, strengthening primary health care, improving education levels, safe water supply and promotion of gender equity.  Key Takeaways:  Topped by: Kerala, Goa, Sikkim, Punjab and Himachal Pradesh secured the top five positions.  Below Average States: It identifies eight States that have scores below the country’s average: they are Assam, Meghalaya, Rajasthan, Chhattisgarh, Madhya Pradesh, Jharkhand, Uttar Pradesh and Bihar.

Link: https://www.thehindu.com/news/national/kerala-tops-in-care-for- children/article32526471.ece President of India’s Greetings on the Eve of Teachers’ Day Question:Consider the following with respect to Sarvepalli Radhakrishnan : (1) His philosophy was grounded in Advaita Vedanta, reinterpreting this tradition for a contemporary understanding. (2) He was also one of the founders of HelpAge India, a non-profit organisation for elderly underprivileged in India. 95 http://www.edristi.in/

Which is/are incorrect? (a) 1 only (b) 2 only (c) Both 1 and 2 (d) None of these Answer:(d) Context:

 The President of India Ram Nath Kovind greeted teachers across the country on the eve of Teachers’ Day. We celebrate the Teachers’ Day on the birth anniversary of Sarvepalli Radhakrishnan.  Facts:  Sarvepalli Radhakrishnan (5 September 1888 – 17 April 1975) was an Indian philosopher, academic, and statesman who served as the first Vice President of India (1952–1962) and the second President of India (1962–1967).  He served as the professor of philosophy at Mysore(1918-21) and Calcutta(1937-41) universities. He was the first Indian to hold a professorial chair at the University of Oxford.  His philosophy was grounded in Advaita Vedanta, reinterpreting this tradition for a contemporary understanding.  He was awarded several high awards during his life, including a knighthood in 1931, the Bharat Ratna in 1954, and honorary membership of the British Royal Order of Merit in 1963.  He was also one of the founders of HelpAge India, a non-profit organisation for elderly underprivileged in India.  Sarvepalli believed that “teachers should be the best minds in the country”. Since 1962, his birthday has been celebrated in India as Teachers’ Day on 5 September every year.

Link: https://pib.gov.in/PressReleaseIframePage.aspx?PRID=1651314#:~:text=In%20a%20m essage%2C%20the%20President,Sarvepalli%20Radhakrishnan. Hampi- Inspired by the past; Going into the Future

Question:Consider the following with respect to Hampi: (1) It is a UNESCO World Heritage Site located in east-central Karnataka. (2) Its name is derived from Pampa which is the old name of the Tungabhadra River on whose banks the city is built. (3) It was the capital of the Bahmani Kingdom in the 14th century. Which is/are correct? (a) 1 and 2 (b) 2 and 3 (c) 1 and 3 (d) 1,2 and 3 Answer:(a)

 Context:

96 http://www.edristi.in/

 Ministry of Tourism has organised a webinar titled Hampi- Inspired by the past; Going into the future under Dekho Apna DeshWebinar series.  Facts:  Hampi: It is a UNESCO World Heritage Site located in east-central Karnataka.  Origin: Its name is derived from Pampa which is the old name of the Tungabhadra River on whose banks the city is built.  Significance: It was the capital of the Vijayanagara Empire in the 14th century.  Trade: By 1500 CE, Hampi-Vijayanagara was the world’s second-largest medieval era city after Beijing and probably India’s richest at that time, attracting traders from Persia and Portugal.  Architecture: The buildings predominantly followed South Indian Hindu arts and architecture dating to the Aihole-Pattadakal styles.They also used elements of Indo- Islamic architecture in the Lotus Mahal, the public bath and the elephant stables.  Important Monuments:  Virupaksha temple( It is dedicated to Virupaksha, a form of Lord Shiva)  Hemkunta Hill, south of the Virupaksha temple contains early ruins, Jain temples and a monolithic sculpture of Lord Narasimha, a form of Lord Vishnu.  Vittal temple built in the 16th Century, is now a World Heritage monument.The columns of the temple are so balanced that they have a musical quality.

Link: https://pib.gov.in/PressReleasePage.aspx?PRID=1649971#:~:text=Ministry%20of%20T ourism%20organises%20a,Dekho%20Apna%20Desh%20Webinar%20Series&text=The %20presenter%20started%20with%20Hampi’s,banks%20the%20city%20is%20built. Indian Railways develops remote-controlled medical trolley ‘MEDBOT’

Question:A remote-controlled medical trolley named ‘MEDBOT’ to help deliver food and medicines to COVID-19 patients is developed by: (a) Zomato (b) Indian Railways (c) Indian Army (d) IIT Delhi Answer:(b)

 Context:  Indian Railways has developed a remote-controlled medical trolley named ‘MEDBOT’ to help deliver food and medicines to COVID-19 patients.  Facts:  It is providing service in the Central Hospital of the Diesel Rail Engine Factory of Indian Railways.  During Corona crisis, apart from providing transport facilities and essential items like foodgrains to people, Indian Railways has also provided facilities for COVID-19 patients.

Link: http://newsonair.com/Main-News-Details.aspx?id=398685

97 http://www.edristi.in/

World’s Largest Tropical Wetland

Question:Pantanal Wetland is found in which of the following countries: (a) Canada (b) Argentina (c) Chile (d) Brazil Answer:(d)

 Context:  Pantanal Wetland is currently suffering its worst fires in more than two decades with nearly 12% of its vegetation reportedly already lost.  Facts:  Pantanal Wetland: It is a natural region encompassing the world’s largest tropical wetland area and the world’s largest flooded grasslands.

 Location: It is located mostly within Brazil but it extends into portions of Bolivia and Paraguay.

 Significance:

 In 2000, part of this ecoregion, the ‘Pantanal Conservation Area’ representing 1.3% of the Brazilian Pantanal was inscribed on UNESCO’s World Heritage List.

 In the same year, a part of Pantanal Wetland was named as UNESCO Biosphere Reserve.The Pantanal Biosphere Reserve is home to a diversity of flora and fauna that is particularly valuable containing at least 4,700 described species. Link: https://www.indiatimes.com/trending/environment/brazils-pantanal-largest-wetland- world-burns- 521641.html#:~:text=The%20Pantanal%2C%20a%20vast%20tropical,its%20vegetation %20reportedly%20already%20lost. State Bank of India’s research report – Ecowrap

Question : According to Ecowrap(State Bank of India’s research report) real GDP of India likely to contract by — (a) 10.9 per cent in financial year 2021 (b) 9.2 per cent in financial year 2021 (c) 8.7 per cent in financial year 2021 (d) 7.2 per cent in financial year 2021 Answer (a) Related facts —

 ’Ecowrap’ is released on 1 september 2020.  It is a State Bank of India’s research report.  Key findings

 After the country’s economy contracted by a record 23.9 per cent in April-June quarter, real GDP for FY21 is expected to shrink by 10.9 per cent. 98 http://www.edristi.in/

 It had earlier estimated real gross domestic product (GDP) at (-) 6.8 per cent for the current fiscal.  The first quarter GDP contraction compares with 3.1 per cent growth in the preceding January-March quarter and 5.2 per cent expansion in the same period a year back.  It estimates Q2 real GDP decline in the range of (-) 12 per cent to (-) 15 per cent, while Q3 GDP is seen between (-) 5 per cent and (-) 10 per cent. Q4 is expected to be in (-) 2 per cent to (-) 5 per cent range.  The report said the country’s GDP growth plunged to 23.9 per cent in Q1 FY21 due to the nationwide lockdown imposed on March 25, 2020, in the wake of the COVID- 19 pandemic and is much worse than market and its estimates.  “Assuming that it remains at 57 per cent of GDP in nominal terms, we will see at least around 14 per cent decline in PFCE(private final consumption expenditure) growth in FY21, as against an average of 12 per cent growth for the nine-year period ended FY20,” the report said.  This indicates an average swing of 26 per cent in current fiscal indicating a consumption washout, it added.

Link: https://www.sbi.co.in/documents/13958/3312806/010920- Ecowrap_20200901.pdf/cf085ebb-ab33-f42a-23e9-25fbb69e6f09?t=1598937075548 Survey of Picodi.com

Question : Picodi.com is an international e-commerce platform that provides discount coupons.It surveyed that an average monthly wage of India is —– (a)437 dollar (b)515 dollar (c)337 dollar (d)217 dollar Answer (a) Related facts

 With an average monthly wage of Rs 32,800 ($437), India has been ranked a lowly 72nd among 106 countries surveyed by Picodi.com, an international e-commerce platform that provides discount coupons.  Switzerland topped the chart, according to a global ranking of average wages prepared by Picodi.com.  Switzerland, where the average wage converted into rupees amounts to Rs 4,49,000 ($5,989), topped the ranking, while in Cuba the average wage is the lowest at Rs 2,700 ($36)  Switzerland was followed by Luxembourg and the United States, whose citizens on an average earn Rs 3,00,900 ($4,014) and Rs 2,64,900 ($3,534), respectively  Others in the top ten list include Denmark ($3,515), Singapore ($3,414), Australia ($3,333), Qatar ($3,232), Norway ($3,174), Hong Kong ($3,024) and Iceland ($2,844).  India, with the average wage of Rs 32,800, outranked countries such as Kazakhstan (Rs 32,700), Brazil (Rs 26,000), or Egypt (Rs 16,400).  Picodi.com analysis team looked into earnings in countries around the world and created a global ranking of average wages.

Link: https://www.picodi.com/ph/bargain-hunting/average-wages

99 http://www.edristi.in/

Report on Fraud & Risk Management in Digital Payments

Question : DSCI stands for — (a) Data Security Council of India (b) Data Security Conference of India (c) Data Settlement Council of India (d) None of the above Answer (a) Related facts

 Data Security Council of India (DSCI) and PayPal India, on Wednesday(26 August 2020) released a study report on Fraud & Risk Management in Digital Payments.  The report attempts to discuss the sophisticated online payment fraud mechanisms, threats in the payment ecosystem, incorporating better fraud prevention strategies, role of upcoming technologies, and recommendations for various stakeholders involved in the payment ecosystem.

Key findings Types of Frauds E-Commerce frauds can be broadly categorised into: Buyer side frauds – fraudulent claims, chargebacks, fake buyer accounts, promotion/coupon abuse, etc. Merchant side frauds – selling counterfeit, non-fulfilment, etc. Cyber security frauds – account takeover, identity theft, card detail theft, triangulation fraud, etc. Key Challenges Fraud detection, enforcement, investigation and legislative challenges Lack of multilevel awareness Cross-industry or expertise collaboration Security is seen as a cost overhead and not essential investments by stakeholders Privacy laws Organised criminal involvement Future Fraud Possibilities Spoofing of Current fraud prevention & detection mechanisms which rely on control parameters like location information; device identifiers like IMEI, MAC address; goods/services identities like SKUs/Barcodes Exploitation of supply chain vulnerabilities at system & human process interchange Exploiting risk transfer controls like buyback, insurance settlement of earlier transactions Link: https://www.dsci.in/content/fraud-and-risk-management-in-digital-payments Japan : Emergency loan for India to fight Covid-19 pandemic

Question : On which date Japan and India established diplomatic relations ? (a) 28th April, 1952 (b) 28th April, 1962 (c) 28th April, 1972 (d) None of the above Answer (a) Related facts

100 http://www.edristi.in/

 Japan announced on Monday(31 August 2020) it will extend a emergency support loan of up to 50 billion yen (almost Rs 3,500 crore) to back India’s response to the Covid-19 crisis, including implementation of health and medical policies and development of hospitals equipped with ICUs.  Japanese ambassador Satoshi Suzuki, and CS Mohapatra, additional secretary in India’s department of economic affairs exchanged notes regarding the provision of the yen loan for the response to the Covid-19 emergency.  A statement issued by the Japanese embassy noted that the Indian government has taken several measures, including health sector reforms, in response to the spread of Covid-19. “This loan provides the necessary funds for emergency response in the fight against Covid-19 in India,” it said.  This is the largest amount of financial assistance announced by any country so far to support India’s response to the Covid-19 crisis, which has had widespread impacts on the economy and health sector.  The terms and conditions of the loan include an interest rate of 0.01% per annum and a redemption period of 15 years, including a four-year grace period.  Suzuki and Mohapatra also exchanged notes for the provision of grant aid worth 1 billion yen under Japan’s Official Development Assistance scheme for India’s Economic and Social Development Programme, which is being implemented by the health ministry.  As of October 2019, the total number of Japanese companies registered in India is 1,454. The number has increased by 13 (0.9% growth), compared to 1,441 in 2018.  The total number of Japanese business establishments in India, as of October 2019, is 5,022, a decrease by 80 (1.6% decline) as compared to 5,102 in 2018.  West Bengal and Haryana showed the largest increase in the number of Japanese companies. There was a significant increase in the number of companies in such sectors as information and communications as well as services.

Link: https://pib.gov.in/PressReleasePage.aspx?PRID=1650053 Karnataka CM flags off Bengaluru to Solapur RORO train

Question:The maiden ‘Roll On Roll Off’ (RORO) train will be operated between Solapur and : (a) Mumbai (b) Chennai (c) Bengaluru (d) Sikanderabad Answer:(c) Context:

 Karnataka CM B S Yediyurappa flagged off first RORO train from Bengaluru to Solapur.  Facts:  The maiden ‘Roll On Roll Off’ (RORO) train between Bengaluru and Solapur in Maharashtra consists of open flat wagon on which trucks with goods are loaded.  It will be operated by the South Western Railway (SWR).

101 http://www.edristi.in/

 The train will take 17 hours to reach its final destination which is at a distance of about 682 kms.  The train will run via Dharmavaram, Guntakal, Raichur and Wadi to reach Bale near Solapur in Maharashtra.  Benefits:  As many as 42 trucks with goods can be carried at time in this train. The RORO service reduces accidents on the road, improves safety, saves fuel, and foreign exchange.  It ensures faster transport of essential goods, perishables, food items and smaller cargo.  While it facilitates the large-scale movement of goods and reduces pollution, the cost of transport is lesser than transport by road.

Link: https://www.thehindu.com/news/national/karnataka/cm-flags-off-roro-train-from- bengaluru-to- solapur/article32478588.ece#:~:text=Chief%20Minister%20B.S.,trucks%20on%20open %20flat%20wagons.

102 http://www.edristi.in/